NOTEPAD
Results
of 200 questions answered correctly

You have reached of 200 points, ( %)

Your time

Question 1 of 200

1. A 49-year-old man complains of pain in his metatarsophalangeal joints and joint deformation. In blood hyperuricemy can be observed. X-ray has revealed metatarsophalangeal joint space narrowing, erosion, periarticular calcification of the both joints, osteoporosis. Microscopy has revealed inflammatory granulomatous reaction surrounding necrotizing masses in the area of the first metatarsophalangeal joint. Choose the most likely diagnosis:

Explanation

krushkrok No21 (2014)

This is a classic description of gout. The end product of the purine nucleotides catabolism in humans and other primates is uric acid (urate) which is excreted in urine. Allopurinol and febuxostat inhibits Xanthine oxidase (XO). Hypoxanthine and Xanthine which is more soluble is excreted in urine. Purine nucleotides (adenine and guanine).  AMP – Adenosine monophosphate; GMP – Guanosine monophosphate

2.

Granulomas containing lymphocytes and macrophages were detected during analysis of skin biopsy material. Among macrophages there are large cells with fat inclusions, which contain microorganisms in spheric packages (Virchow’s cells). The following disease is based on the described type of hypersensitivity:

Explanation

     Leprosy (Hansen’s disease) is a chronic generalized infectious disease, which involves mainly the ectoderm components (the skin, mucouc membranes and the peripheral nervous system), lymphatic nodes, liver, spleen, bones and endocrine glands. The causative agent is acid proof and alcohol-resistant Mycobacteria leprae. Three types of leprosy are distinguished: lepromatous; tuberculoid; undifferentiated (the first 2 are the main forms). In tuberculoid leprosy, the epidermis contains confluent granulomas composed of macrophages, plasma cells and leprous Virchow’s cells. Leprous Virchow’s cells (or leprosy/hansen’s cells) refer to large foamy macrophages within fatty vacuoles containing leprous mycobacterium.

3. A patient with suspected necrosis of the upper abdominal cavity organs was delivered to a surgical department. This condition is associated with acute circulatory disturbance of the following vessel:

Explanation

image

The abdominal cavity is divided into 3 storeys:

Upper storey:

Borders – superior: inferior surface of the diaphragm

                - Inferior: transverse mesocolon

Contents – hepatic bursa, pregastric bursa, omental bursa, liver, stomach, gall bladder, spleen, superior poles of kidneys, adrenal glands, superior part of the duodenum, abdominal aorta and inferior vena cava

Celiac trunk: arises from the upper portion of abdominal aorta at the level of Th12. It appears as thick but short (1-2cm long) trunk that gives off 3 branches (this division is called the Haller’s tripod): the left gastric, common hepatic and the splenic arteries that supply the stomach, liver and spleen respectively.

4. Name the drug that inhibits excretory function of pancreas during treatment of acute pancreatitis:

Explanation

α2-macroglobulin and α1-antitrypsin are protease inhibitors which can inhibit the pancreatic proteases and prevent further tissue protein breakdown.
5.

An 18-year-old patient has developed candidiasis after the case of pneumonia treated with β-lactam antibiotic. What anti-mycotic agent should be prescribed?

Explanation

Infectious Diseases caused by fungi are called MYCOSES and they are often chronic in nature. Fluconazole is a member of the triazole class of antifungal products. It inhibits the synthesis of fungal membrane Ergosterol in the same manner as Ketoconazole.

image

Fluconazole is effective against most forms of mucocutaneous candidiasis. It is administered orally or intravenously.

6. During autopsy of a 9-month-old girl’s body, who died due to severe pneumonia complicated with sepsis, lack of thymus is observed. In the lymph nodes the lymphoid follicles and cortical substance are absent; follicles of spleen are reduced in size with no light zones and plasma cells. What is the cause of such structural changes?

Explanation

There is a certain correlation between age of an individual and thymus activity. In neonates, it already appears to have a considerable mass of 13.3g on the average. Most intensive growth occurs during first 3 years of life, when the gland doubles in size. Having reached the maximum weight (about 26 – 30g), the thymus retains it until 20 years of life. After 20 years of age, the thymus parenchyma experiences gradual involution and is substituted with fat tissue. After 50 years of age, the fat constitutes 90% of gland mass. Therefore, for a 9-month-old girl, lack of thymus indicates that the thymus did not develop at all, hence, Thymus agenesis.
7. A patient with femoral neck fracture, who for a long time had to remain in bed in a forced (supine) position, has developed dark-brown lesions along the backbone; soft tissues are swollen, in the areas of maceration there is a foul-smelling liquid. Name the clinicopathologic type of necrosis:

Explanation

Sustained or repeated pressure on skin over bony prominences can cause ischemia and pressure sores. These are common in patients over 70 years old who are confined to hospital, especially those with a fracture of the neck of femur. The morbidity and mortality of those with deep ulcers is high.

Causes:

*prolonged immobility and recumbency

*vascular disease (atherosclerosis)

*neurological disease causing diminished sensation.

*malnutrition,severe systemic disease and general debility.

Clinical Features: The sore begins as an area of erythema which progresses to a superficial blister or erosion. If pressure continues, deeper damage occurs with the development of a black eschar which  when removed or shed reveals a deep ulcer, often colonized by Pseudomonas aeruginosa. The skin overlying the sacrum, greater trochanter, ischial tuberousity, the heel and the lateral malleolus is especially at risk.

8.

A woman poisoned with unknown substance was hospitalised in a toxicological department. What group of drugs can be administered to decrease absorption and introduction of the poison to her body?

Explanation

The main purpose of detoxification therapy is to reduce poison absorption and to enhance the removal of poison. For the reduction of absorption the use of adsorbents like activated charcoal (carbo activatus), enterosgel are effective.

Adsorbents are insoluble fine powders which have a large active surface capable of fixing irritating and poisonous substances dissolved in water and gases, thus preventing their absorption in the GI tract and protecting receptors. Carbo activates is administered orally in the form of tablets or non-dosed powder. It is used in acute poisonings (as a universal antidote), as well as in enterocolitis, enteritis, dyspepsia, meteorism etc.

9. A patient after disrupted cerebral circulation has developed paralysis. Choose the anticholinesterase drug to be prescribed in this case:

Explanation

Acetylcholine esterase (AChE) is an enzyme that specifically cleaves acetylcholine to acetate and choline and thus, terminates its actions. Inhibitors of AChE indirectly provide a cholinergic action by prolonging the lifetime of acetylcholine produced endogenously at the cholinergic nerve endings. This results in the accumulation of Acetylcholine in the synaptic space by blocking the step No5.

image

Therefore, these drugs can provoke a response at all cholinoreceptors in the body, including both muscarinic and nicotinic receptors of the autonomic nervous system as well as at neuromuscular junctions and in the brain.

Neostigmine (proserine) is a synthetic compound that is also a carbamic acid ester and it reversibly inhibits acetylcholine esterase (AChE).

10.

A 50-year-old woman is being treated for shingles in a neurology unit. What reactivated virus causes this disease?

Explanation

Varicella-Zoster virus (VZV):

image

Varicella (chicken pox) is the primary disease; Zoster (shingles) is the recurrent form. The virus is transmitted by respiratory droplets and by direct contact with the lesions. VZV infects the mucosa of the upper respiratory tract, then spreads via the blood to the skin, where the typical vesicular rash occurs. After the host has recovered the virus becomes latent, probably in the dorsal root ganglia. Later in life, frequently at times of reduced cell-mediated immunity or local trauma, the virus is activated and causes the vesicular skin lesions and nerve pain in Zoster.

Immunity following Varicella is lifelong: a person gets Varicella only once, but Zoster can occur despite this immunity to Varicella. Zoster usually occur only once also.

Acyclovir, famciclovir, valacyclovir are drugs that can be used for treatment.

11.

During examination of a patient a doctor should use anatomical division of anterior abdominal wall into regions for more precise diagnostics. How many such regions can abdomen be divided into?

Explanation

image

Anterolateral abdominal wall is the muscular aponeurotic formation limited by the abdominal wall from the anterior and lateral sides. There are two horizontal orientation lines and two vertical orientation lines that divides it into 9 regions, namely:

·        Right hypochondriac region

·        Proper epigastric region

·        Left hypochondriac region

·        Right lateral abdominal region

·        Umbilical region

·        Left lateral abdominal region

·        Right inguinal (ilioinguinal) region

·        Hypogastric (suprapubic) region

·        Left inguinal (ilioinguinal) region.

12. During surgery performed in abdominal cavity a surgeon located ligament of liver stretching from anterior abdominal wall (navel) to inferior surface of liver. What ligament is it?

Explanation

Round ligament of liver (ligamentum teres hepatis): it runs from the visceral surface to the navel; it is an obliterated peritoneum–enfolded umbilical vein. The orifice of the vein may remain partially patent (open) and thus the vein can be used for infusion of drugs and radiopaque agents. The umbilical vessels closure occurs during the first week of life; the umbilical vein transforms into peritoneum–enfolded round ligament of liver. The umbilical arteries also close and transform into the medial umbilical ligaments. The single umbilical vein is responsible for carrying oxygenated blood from placenta to fetus in fetal circulation.

13.

A 9-year-old boy has acute onset of disease: sore throat, body temperature rise up to 39, 5oC ; on the second day diffuse skin rash was detected all over his skin except for nasolabial triangle. On examination of oral cavity: crimson tongue, \\\"flaming pharynx\\\", necrotic tonsillitis. What diagnosis is the most likely?

Explanation

Scarlet fever is an acute infectious disease accompanied by local inflammatory changes mainly in the pharynx and typical generalized rash.

* Local changes: primary scarlatnic affect/complex – catarrhal or necrotic tonsillitis. Catarrhal tonsillitis: hyperemia of pharynx (flaring pharynx or burning faucet). Necrotic tonsillitis: coagulative necrosis and ulceration.

* General changes are first of all RASH. The face is also involved, but usually a small area about the mouth (nasolabial triangle) remains relatively unaffected to produce a circumoral pallor. Tongue is beefy red/crimson red and glistening, strawberry tongue.

14. During narcosis a patient developed a risk of cerebral edema. What drug should be administered in this case?

Explanation

IMG_9896

Bumetanide, furosemide, torsemide, ethacrynic acid are diuretics that have their major action on the ascending limb of the loop of henle (loop diuretics). These drugs are useful in emergency situation which calls for a rapid, intense dieresis (forced dieresis). Loop or high-ceiling diuretics inhibit the cotransport of Na+/K+/2Cl- on the luminal membrane in the ascending limb of the loop of henle.

15.

A 27-year-old patient with injury to the neck has lost approximately 30% of the blood volume. The patient’s condition is severe: blood pressure is 60/40 mm Hg, heart rate is 140/min., respiratory rate is 30/min., conscious. Characterize the condition of the patient’s circulatory system:

Explanation

Hypovolemic shock is due to excessive loss of sodium-containing fluid (e.g. blood, sweat), causing hypotension and multiorgan failure. Massive blood loss is the most common cause. Loss of >20% of blood volume (approx. 1000mL) results in shock. Common causes of external blood loss includes penetrating trauma and gastrointestinal bleeding.

Decreased cardiac output - due to decreased volume of blood; increased peripheral vascular resistance – due to vasoconstriction of arterioles from catecholamines, vasopressin and angiotensin II, which are released  in response to the decreased cardiac output.

Clinical findings: cold, clammy skin due to vasoconstriction of skin vessels, hypotension along with a rapid, weak pulse; urine output is decreased because of decreased renal blood flow and glomerular filtration rate.

Treatment: fluid replacement (e.g. normal saline, blood transfusions)

16. A soldier with explosion-caused trauma was delivered to a hospital. Examination revealed his tympanic membrane to be intact. What defense reflex prevented the tympanic membrane from rupturing?

Explanation

Two skeletal muscles are attached to the ossicles in the middle ear: tensor tympani and stapedius. Stapedius is the smallest skeletal muscle in the human body. Its tendon is inserted into the posterior surface of the neck of stapes. The tendon of tensor tympani is inserted on manubrium of malleus, which in turn is attached to the tympani membrane.

Tympanic reflex is an attenuation reflex characterized by involuntary contraction of tensor tympani and stapedius muscles in response to a loud noise. When both the muscles contract, manubrium of malleus moves inward and stapes is pulled outward. These two actions results in stiffness of auditory ossicles, so that the transmission of sound is decreased. This reflex protects the tympanic membrane from being ruptured by loud sound.

17.

A person with the fourth blood group (genotype IAIB) has in erythrocytes both antigen A controlled by allele IA and antigen B controlled by allele IB. This phenomenon is an example of the following gene interaction:

Explanation

Codominance: there are several conditions where two or more alleles do not show complete dominance or recessiveness due to the failure of any allele to be dominant in the heterozygous condition. In most cases the heterozygote has a prototype, which is intermediate between the homozygous dominant and recessive conditions. Codominance refers to instances in which two alleles are expressed independently in the heterozygote. The human AB blood group provides a classic example of codominant alleles.

18.

An 18-year-old woman has body disproportion, wing-like folds on the skin of her neck, underdeveloped ovaries, nuclei of her buccal epithelium cells have no Barr bodies. Dermatoglyphics method revealed that her adt angle is 66o. What provisional diagnosis can be made in this case?

Explanation

FullSizeRender (1)

krushkrok No105 (2014)

These characterize Turner’s syndrome (XO). It can be complete monosomy (45,XO) or mosaicism (e.g. 45,XO/46,XX). Barr body is an inactive X-chromosome. A normal female has one barr body XX; so in turner’s syndrome, there is no barr body because the only X present is the active one (45 XO).

19.

A patient diagnosed with acute dysentery has been treated for 3 days in an infectious diseases hospital. On admission there were complaints of high temperature, stomachache and fluid excrements with mucus as often as 8-10 times a day. What sample should be taken for analysis?

Explanation

Dysentry refers to bloody diarrhea  with mucus. It can be caused by a variety of micro-organism. It is a sign of large bowel disease. It can also be of infectious origin. Diarrhea could be a sign of infection, laxative abuse, inflammatory bowel disease. Diarrhea- more than 250g of stool per day. Acute diarrhea is defined as less than 3 weeks, chronic diarrhea over 4 weeks.

Important screening tests:

*Fecal smear for leucocytes.

*Stool osmotic gap.

20. A patient visited a dentist to extract a tooth. After the tooth had been extracted, bleeding from the tooth socket continued for 15 minutes. Anamnesis states that the patient suffers from active chronic hepatitis. What phenomenon can extend the time of hemorrhage?

Explanation

Substances necessary for coagulation or clotting of blood are called clotting factors. Fibrinogen is the factor I of the thirteen clotting factors. Most of the clotting factors are proteins in the form of enzymes synthesized in the liver.

In chronic hepatitis (i.e. relatively prolonged course of inflammation of the liver), the protein-synthetic function of the liver is impaired. Meaning, there is a marked decrease of clotting factors which results in uncontrolled and protracted hemorrhage.

21.

Name the halogen-containing antiseptic with fungicidal properties, which is used to treat dermatomycosis:

Explanation

Antimicrobial agents are drugs for the treatment and prevention of infectious diseases. They are divided into disinfectants, antiseptics and chemotherapeutics. Disinfectants realize their antimicrobial properties in the environment outside the body. Antiseptics act on the surface of the body. Chemotherapeutics produce an antimicrobial effect inside the body.

Antiseptics could be inorganic or organic.

·        Inorganic substances: halogens, oxidizing agents, metallic salts, acids and alkalis

·        Organic substances: aldehydes, alcohols, phenol derivatives, dyes, detergents, nitrofuran derivatives

Halogens: iodine alcohol solution – effects of iodine are based on the interaction between atoms of halogen and proteins resulting in halogenization and oxidation of proteins. It has bactericidal, fungicidal and irritative actions. Indications – processing of small cuts of the skin, dermatomycoses, processing of the surgery skin area and surgeon’s hands.

Other halogens include solution of lugol, iodinol, ioddiccerinum, chlorinated lime, chloramine B, chlorhexidine etc.

22.

Due to severe pain syndrome a patient was prescribed a narcotic analgesic. Name this drug:

Explanation

Pain is an unpleasant sensation that can be either acute or chronic and is a consequence of complex neurochemical processes in the peripheral and central nervous system.

Analgesics are drugs reversibly and selectively inhibiting pain in the body without significant changing of consciousness. Opoid (narcotic) analgesics are the drugs that relieve intense pain which mimic the action of endogenous opiopeptides and may cause drug dependence.

Strong agonists of opoid receptors:

·        Natural compounds: morphine hydrochloride, codeine phosphate

·        Synthetic compounds: fentanyl, promedol

Acute poisoning with morphine: signs – state of sleep, unconsciousness, miosis, bradycardia, cheyne-stokes breathe, retention of urination, spasm of the intestine and bowel. Emergency help:

·        Lavage of stomach by 0.5% sol. of potassium permanganate

·        Naloxone, IV (an antagonist of narcotic analgesics)

·        Atropine (for a decrease in the vagal action of morphine)

23.

During pathomorphological kidney investigation of a patient, who for a long time had been suffering from osteomyelitis and died from progressing renal failure, the following was revealed: deposits of homogeneous eosinophilic masses in glomerular mesangium, arterial and arteriolar walls, and stroma, which became red when stained with Congo red. What pathological process is this?

Explanation

Amyloidosis is the term used for a group of diseases characterized by extracellular deposition of fibrillar proteinaceous substance called amyloid.

Amyloidosis of the kidneys is the most common and most serious because of ill-effects on renal function. Cut surface of kidney is pale, waxy and translucent. In the glomeruli, the deposits initially appear on the basement membrane of the glomerular capillaries but later extend to produce luminal narrowing and distortion of the glomerular capillary tuft. There is also narrowing of the small arterioles and venules and consequent ischemic effects.

The deposits in the kidneys are found in most cases of secondary amyloidosis. The histologic diagnosis of amyloid is based almost entirely on its staining characteristics: Congo red – all types of amyloid have affinity for Congo red stain.

24. A microslide presents a tissue with spherical cells, each of them containing a large fat drop covered with thin cytoplasm layer in its center. Nucleus is compressed and situated at the cell periphery. What tissue is it?

Explanation

There are two types of adipose tissue: white (unilocular) and brown (multilocular). White adipose tissue is the predominant type in adult humans. Brown adipose tissue is present in humans during fetal life but diminishes during the first decade after birth.

image

In the late stage of differentiation of white adipose tissue, the cells increase in size and become more spherical. Small lipid droplets coalesce to form large lipid vacuoles that occupy the central portion of the cytoplasm. Eventually, the lipid mass compresses the nucleus to an eccentric position, producing a signet-ring appearance in H & E preparations.

25.

To treat rheumatoid arthritis a 65-year-old woman was prescribed an immunosuppressive hormonal drug as a part of her complex therapy. Name this drug:

Explanation

Cortisol is the principal human glucocorticoid. Glucocorticoids are adrenal steroids with a prevalent action on metabolism and inflammation.

Short-Acting Glucocorticoids: Hydrocortisone Acetate.                           

 Intermediate  Acting: Prednisolone.

Long Acting: Dexamethasone.

   Glucocorticoids are among the most potent anti-inflammatory,anti-alergic,immunosuppressive,anti-shock drug. They inhibit all three (3) stages of inflammation.

   Indications include: Collagenosis, Severe Rheumatism, Arthritis, Arthrosis, Bronchial Asthma, Allergic Diseases of the skin and mucous membranes, Autoimmune Diseases etc.

26.

An experiment was aimed at testing flexor reflex in a spinal frog, which was initiated by simultaneous stimulation with isolated pre-threshold electrical impulses. The frequency of those impulses was such, that the reflex occurred. What process in the nerve centers can be observed during this experiment?

Explanation

image

Summation is the fusion of effects or progressive increase in the excitatory postsynaptic potential in post synaptic neuron when many presynaptic excitatory terminals are stimulated simultaneously or when single presynaptic terminal is stimulated repeatedly. Summation is one of the properties of synapse and is of two types:

*Temporal Summation occurs when one presynaptic terminal is stimulated repeatedly.

*Spatial Summation occurs when many presynaptic terminals are stimulated.

27. Microelectrode analysis of nerve fiber bioelectrical activity revealed, that its membrane potential equals 90 mV. Its initial rest potential was 85 mV. What process occurs in this case?

Explanation

Hyperpolarization is a change in a cell’s membrane potential that makes it more negative. It is the opposite of depolarization. It inhibits action potentials by increasing the stimulus required to move the membrane potential to the action potential threshold. Hyperpolarization is often caused by efflux of K+ through K+ channel or influx of Cl- through Cl- channels. Membrane potential is the electrical potential difference (voltage) across the cell membrane (between inside and outside of the cell) under resting condition.

In the question, the initial rest potential is 85mV. But it increased to 90mV i.e. either there is an efflux of K+ (loss of K+) or influx of Cl-, which has increased the electronegativity inside the cell.

On the other hand, if it were to be depolarization, we’ll have something like: rest potential 85mV; then membrane potential 80mV

28.

Parkinson’s disease is caused by disruption of dopamine synthesis. What brain structure synthesizes this neurotransmitter?

Explanation

Parkinson disease is associated with a loss of dopaminergic neurons in the substantia nigra. Parkinsonism is a progressive neurological disorder of muscle movement, characterized by tremors, muscular rigidity, bradykinesia (slowness in initiating and carrying out voluntary movements) and postural and gait abnormalities. Most cases involve people over the age of 65.

The disease is correlated with destruction of dopaminergic neurons in the substantial nigra with a consequent reduction of dopamine actions in the corpus striatum, parts of the brain’s basal ganglia system that are involved in motor control.

The substantial nigra, part of the extrapyramidal system is the source of dopaminergic neurons that terminate in the neostriatum.

Treatment: drugs – levodopa, carbidopa, selegiline, rasagiline

So far, levodopa has been the only drug tested on Parkinson.

29.

Determining a patient’s blood group with monoclonal test-reagents revealed positive agglutination reaction to anti-A and anti-B reagents, and negative reaction to anti-D. What blood group does this patient have?

Explanation

image

Blood group O(I): no antigens, therefore no agglutination.

Blood group A(II): A antigen, agglutinate with blood group B(III) and O(I).

Blood group B(III): B antigen, agglutinate with blood group A(II) and O(I).

Blood group AB(IV): A and B antigen, agglutinate with all blood groups. No antibody.

Blood group name is determined by the antigen present on RBC, but the patient has an opposite antibody. So whenever, the antibody corresponds to the antigen, there is agglutination. Since O does not have any antigen, no agglutination can occur.

30. A 53-year-old man suffering from diabetes mellitus has developed a painful conical induration, bluish-red with yellow center, on the skin of his neck. Such changes are characteristic of:

Explanation

A furuncle or boil is a focal suppurative inflammation of the skin and subcutaneous tissue. They may be solitary or multiple or recur in successive crops. Furuncles are most frequent in moist, hairy areas such as the face, axillae, groin, legs, neck and mammary folds. Beginning in a single hair follicle, a boil develops into a growing and deepening abscess that eventually “comes to a head” by thinning and rupturing the overlying skin. It is a cutaneous complication of poorly treated or managed diabetes mellitus.

31.

Biochemical analysis of an infant’s erythrocytes revealed evident glutathione peroxidase deficiency and low concentration of reduced glutathione. What pathological condition can develop in this infant?

Explanation

Erythrocytes are totally dependent on the pentose phosphate pathway for their supply of NADPH because, unlike other cell types, erythrocytes do not have an alternate source for this essential coenzyme. If glucose 6-phosphate dehydrogenase (G6PD) is compromised in some way, NADPH levels will fall and oxidized glutathione cannot be reduced. As a result, hydrogen peroxide will accumulate, threatening membrane stability and cause red cell lysis – hemolysis.

Reduced glutathione present in most cells, can chemically detoxify hydrogen peroxide (H2O2). This reaction, catalyzed by the selenium-requiring glutathione peroxidase, forms oxidized glutathione which no longer has protective properties. The cell regenerates reduced glutathione in a reaction catalyzed by glutathione reductase, using NADPH as a source of reducing electrons. Thus, NADPH indirectly provides electrons for the reduction of hydrogen peroxide.

32.

In winter a 3-year-old child has sharp rise of body temperature up to 40oC . Hemorrhagic rash is observed on the skin and mucosa. Bean-shaped gram-negative microorganisms situated in pairs are detected in the blood. What provisional diagnosis can be made?

Explanation

Neisseria diplococcic are gram negative cocci that resemble paired kidney beans/coffee grains. The genus Neisseria contains two important human pathogens: Neisseria meningitides and Neisseria gonorrhoeae. Portal of entry is respiratory and genital tracts respectively. Meningococcal infection is an acute infectious process which has 3 main forms: nasopharyngitis; purulent meningitis and meningococcemia.

Meningitis is an acute or chronic inflammatory process chiefly affecting the pia and arachnoid mater (brain tunics) and cerebrospinal fluid. Meningococcal meningitis is characterized by the hyperemia of the pia mater, saturated with dull serous exudates during the first days of the disease.

33. A 50-year-old man, who has been suffering from chronic hepatic failure for several years, has developed ascites. What is the main mechanism of this disorder development?

Explanation

image

The major complications of the increased portal vein pressure and the opening of collateral channels are ascites, splenomegaly and the formation of portosystemic shunts with bleeding from esophageal varices.

Ascites occur when the amount of fluid in the peritoneal cavity is increased and is a late-stage manifestation of cirrhosis, hepatic failure which results in portal hypertension. Portal hypertension is characterized by increased resistance to flow in the portal venous system and sustained portal vein pressure above 12mmHg (normal 5-10mmHg). Portal hypertension can be caused by a variety of conditions that increase resistance to hepatic blood flow, including pre-hepatic, posthepatic and intrahepatic obstructions.

Clinical findings in ascites: abdominal distention with a fluid wave; increased risk for spontaneous bacterial peritonitis.

34. A 30-year-old man has sustained an injury to his thorax in a traffic incident, which caused disruption of his external respiration. What type of ventilatory difficulty can be observed in the given case?

Explanation

Restrictive respiratory disease is the abnormal respiratory condition characterized by difficulty in inspiration. Expiration is not affected. It is characterized by the reduced expansion of the lung parenchyma accompanied by the decreased total lung capacity. It includes poliomyelitis, myasthenia gravis, Paralysis of diaphragm or pneumothorax.

Pneumothorax is the presence of air in pleural space (extrapulmonary – lung is not affected). Intrapleural pressure which is always negative, becomes positive in pneumothorax and it causes collapse of lungs. It is caused by the entry of air into the pleural cavity because of damage of chest wall or lungs during accidents, bullet injury or stab injury. But in the question, the injury was to the thorax alone, the lungs was not affected therefore, it is extrapulmonary (restrictive) ventilatory impairment.

35.

A patient is diagnosed with cardiac infarction. Blood test for cardiospecific enzymes activity was performed. Which of the enzymes has three isoforms?

Explanation

 Isoenzyme is any of several forms of an enzyme that all catalyze the same reaction but may differ in reaction rate, inhibition by various substances, electrophoretic mobility or immunologic properties. Examples of important enzymes with isoenzymic forms are:

·        Lactate dehydrogenase: LDH1, LDH2, LDH3, LDH4, LDH5, (5 isoenzymic forms).

·        Creatine kinase (CK): CK-MM; CK-MB; CK-BB (3 isoenzymic forms)

36. A patient is diagnosed with chronic atrophic gastritis attended by deficiency of Castle’s intrinsic factor. What type of anemia does the patient have?

Explanation

Pernicious anemia or addison’s anemia is the anemia due to deficiency of vitamin B12. It is due to atrophy of the gastric mucosa (atrophic gastritis) because of autoimmune destruction of parietal cells. The gastric atrophy results in decreased production of intrinsic factor and poor absorption of vitamin B12, which is the maturation factor for red blood cells (RBC). RBCs are larger and immature with almost normal or slightly low hemoglobin level. Synthesis of hemoglobin is almost normal in this type of anemia. So, cells are macrocytic and normochromic/hypochromic.

37. A pregnant woman was detected with IgM to rubella virus. An obstetrician-gynecologist recommended therapeutic abortion due to the high risk of teratogenic affection of the fetus. Detection of IgM was of great importance as it is these specific immunoglobulins that:

Explanation

High IgM level usually indicate an acute or primary infection because, it is the first antibody to appear in response to initial exposure to an antigen. Secondary or reinfection show an increase in IgG. IgM antibodies appear early in the course of an infection, this makes it useful in the diagnosis of infectious diseases. Demonstrating IgM antibodies in a patient’s serum indicates recent infection or in a neonate’s serum, it indicates intrauterine infection.

38.

Obesity is a common disease. The aim of its treatment is to lower content of neutral fats in the body. What hormone-sensitive enzyme is the most important for intracellular lipolysis?

Explanation

Hormone-sensitive triacylglycerol lipase is so named because its activity is mostly controlled by hormones. Several hormones such as epinephrine (most effective), norepinephrine, glucagon, thyroxine, ACTH etc. – enhance the activity of adenylate cyclase and thus increase lipolysis. The complete degradation of triacylglycerol (TAG) to glycerol and free fatty acids is known as lipolysis.

Lipase is present in an inactive form and is activated (phosphorylated) by a cAMP dependent protein kinase to lipase. On the other hand insulin decreases cAMP levels and thereby inactivates lipase. Caffeine promotes lipolysis by increasing cAMP levels through its inhibition on phosphodiesterase activity.

39.

A 40-year-old woman was diagnosed with glomerulonephritis based on her clinical symptoms and the results of urine analysis. Anamnesis states chronic tonsillitis. What microorganisms are the most likely cause for her kidney damage?

Explanation

 Acute poststreptococcal glomerulonephritis: most frequently seen in children. Occurs approximately 2 weeks after group A streptococcal infection of pharynx or skin. Resolves spontaneously. Type III hypersensitivity reaction (Immune complex). Presents with peripheral and periorbital edema, cola-coloured urine, hypertension. On immunofluorescent microscopy: granular appearance due to IgG, IgM and C3 deposition along glomerular basement membrane and mesangium. On electron microscopy: subepithelial immune complex humps. On light microscopy: glomeruli enlarged and hypercellular.

40.

A man is suffering from diarrhea. In summer he spent his vacation in the south at the sea coast. Bacteria with the following properties were detected in his feces: gram-negative curved mobile monotrichous bacilli that do not produce spores or capsules. They are undemanding to nutrient medium but require alkaline reaction (рН 8,5-09,5). Described are the agents of the following enteric infection:

Explanation

Vibrio cholera is the cause of cholera. It is curved, comma-shaped, gram negative rods. It is transmitted by fecal contamination of water and food, primarily from human sources. Clinically, watery diarrhea in large volumes is the hallmark of cholera. There are no red blood cells or white blood cells in the stool. Rice-water stool is the term often applied to the non-bloody effluent. Grows in an alkaline media. Dark-field and phase contrast microscopy have been used for screening fecal specimens for the presence of V. cholera. With these techniques, liquid stools are microscopically examined for the presence of organisms with typical darting (“shooting star”) mobility [analogous to ‘hanging drop’].

41.

Cholesterol content in blood serum of a 12-year-old boy is 25 mmol/l. Anamnesis states hereditary familial hypercholesterolemia caused by synthesis disruption of receptor-related proteins for:

Explanation

Type IIa (familial hyperlipoproteinemia: ↑LDL and cholesterol. Autosomal dominant; due to absent or defective LDL receptors.

Type I:  ↑chylomicrons, triacylglycerol (TAG), cholesterol. Autosomal recessive; due to lipoprotein lipase deficiency or altered apolipoprotein C-II.

Type IV: ↑very low density lipoprotein (VLDL) and TAG. Autosomal dominant; due to hepatic overproduction of VLDL.

Type IIb: similar to Type IIa, except that VLDL is also increased and VLDL is normal for IIa.

42. A 3-year-old child has eaten some strawberries. Soon he developed a rash and itching. What was found in the child’s leukogram?

Explanation

This is an example of type I hypersensitivity reaction considering the time taken for the symptoms (rash and itching) to manifest and the expression of eosinophilia.

Neutrophilic leukocytosis: bacterial infection

Lymphocytosis: viral and bacterial infection (e.g. whooping cough, tuberculosis)

Monocytosis: chronic inflammation (tuberculosis); autoimmune disease

43.

A patient has been found to have a marked dilatation of the saphenous veins in the region of anterior abdominal wall around the navel. This is symptomatic of pressure increase in the following vessel:

Explanation

image

Portal hypertension:

Pathogenesis: resistance to intrahepatic blood flow due to intrasinusoidal hypertension. Anastomoses between portal vein (vena porta hepatis) tributaries and the arterial system. Complications: ascites, periumbilical venous collaterals (caput medusa); esophageal varices; congestive splenomegaly; hemorrhoids

↑in blood pressure in portal vein → portal hypertension

44.

A woman complains of visual impairment. Examination revealed obesity in the patient and her fasting plasma glucose level is hyperglycemic. What diabetes complication can cause visual impairment/blindness?

Explanation

Microangiopathy is a chronic complication of diabetes mellitus (fasting hyperglycemia) affecting small blood vessels and manifested as retinopathy (vision impairment), nephropathy, neuropathy. Macroangiopathy affects larger vessels causing symptoms such as gangrene, ulceration, transient ischemic attack.
45.

Prolonged treatment of hypothyroidism has caused general dystrophy, dental caries, tachycardia, tremor of extremities. What drug is the cause of these side effects?

Explanation

There are two active forms of thyroid hormone: thyroxine (T4) and triiodothyronine (T3, which is the more potent form). Levothyroxine (L-thyroxine) and triiodothyronine are the drug forms respectively. Thyroid hormone therapy is used for the treatment of hypothyroidism (myxedema), including myxedema coma. Its therapeutic uses include hypothyroidism, diffuse non-toxic goiter, hashimoto’s thyroiditis. Thyroid hormone is used in patients with simple goiter and in patients with nodular goiter who are deficient in the secretion of thyroid hormone. If thyroid hormone therapy is begun early after birth in hypothyroid infants, the consequences of cretinism are prevented.

46.

This year influenza epidemic is characterised by patients’ body temperature varying from 36, 9o C to 37, 9oC . Such fever is called:

Explanation

Normal 36.5 – 36.8oC

Subfebrile 36.9 – 37.9oC

Febrile 38.0 – 38.9oC

Pyretic 39.0 – 40.9oC

Hyperpyretic ≥41oC

47. A 25-year-old patient complains of increasing pain in his leg muscles occurring during walking and forcing him to make frequent stops. Objectively: skin of legs is pale, no hair-covering, toenails are with trophic changes, no pulsation of pedal arteries. The most probable cause of these changes is:

Explanation

Characteristics of ischemic feet includes: cold to the touch; no palpable foot pulses; atrophic; hairless; tender; painful. This can be compared to neuropathic feet which is: warm to the touch; there is palpable foot pulses, but the skin is dry. These two types of feet can be seen commonly in diabetic patients.

48.

A 30-year-old patient’s blood test revealed the following: erythrocyte count is 6 · 1012 /l, hemoglobin is 10,55ммоль/l. Vaquez’s disease was diagnosed. Name the leading part of pathogenesis:

Explanation

Hemoblastoses refer to neoplastic clonal diseases, arising from hemopoietic organs. Erythrocytes which originate from the hemopoietic organ can be greatly increased (erythrocytosis) in myeloproliferative disorders. Myeloproliferative disorders includes chronic myeloleukemia, polycythemia vera (erythemia, Vaquez disease, Osler’s disease, osler-vaquez disease) etc. Vaquez disease is a primary form of erythrocytosis.

49. A 6-year-old child suffers from delayed growth, disrupted ossification processes, decalcification of the teeth. What can be the cause?

Explanation

image

Vitamin D is a fat soluble vitamin. Cholecalciferol (Vit. D3) is found in animals. It is regarded as a sunshine vitamin. Calcitriol is the biologically active form of Vit. D. Calcitriol acts at 3 different levels (intestine, kidney and bone) to maintain plasma calcium.

It increases the intestinal absorption of calcium and phosphate.

In the osteoblasts of bone, it stimulates calcium uptake for deposition as calcium phosphate. Thus, calcitriol is essential for bone formation.

Calcitriol is also involved in minimizing the excretion of calcium and phosphate through the kidney, by decreasing their excretion and enhancing reabsorption.

image

Deficiency of Vit D causes rickets in children and osteomalacia in adults. Rickets in children is characterized by bone deformities due to incomplete bone mineralization, resulting in soft and pliable bones and delay in teeth formation. The weight bearing bones are bent to form bow legs.

50. A patient consulted a dentist about restricted mouth opening (trismus). Anamnesis states a stab wound of the lower extremity. What infection may cause these symptoms?

Explanation

Tetanus is caused by Clostridium tetani that secretes a toxin called tetanospasmin. Findings: spasticity, risus sardonicus (raised eyebrows and open grin) and lockjaw (trismus); toxin prevents release of inhibitory (GABA and glycine) neurotransmitters from Renshaw cells in spinal cord. It can be prevented with tetanus vaccine.

51.

A patient has damaged spinal cord white matter in the middle area of the posterior white column, disrupted proprioceptive sensibility of the lower limb joints and muscles. What fibers are affected?

Explanation

image

Fasciculus gracilis (tract of Goll) and Fasciculus cuneatus (tract of Burdach): fasciculus gracilis and cuneatus are together called ascending posterior column tracts. These tracts are formed by the fibers from posterior root ganglia. Thus, both tracts are constituted by the fibers of first order neurons of sensory pathway. Functions: fine (epicritic) tactile sensation; tactile localization; tactile discrimination; sensation of vibration; conscious kinesthetic sensation; stereognosis (ability to recognize the known objects by touch with closed eyes).

Fasciculus gracilis contains the fibers from lower extremities and lower parts of the body. Fasciculus cuneatus contains fibers from upper part of the body.

52.

In an elderly person the change in heart force and vessels physical properties were detected; they can be clearly observed on graphic recording of carotid pulse waves. What method was applied?

Explanation

Sphygmography: pulsation of the vascular wall is recorded as a curve (sphygmogram) by an apparatus called sphygmograph. Sphygmograph is an instrument for graphically recording the form, strength and variations of the arterial pulse. Special pick-ups convert mechanical oscillations of the vascular wall into electric pulses, which are amplified and recorded by an electrocardiograph.

Direct and Indirect sphygmography is distinguished. Direct sphygmography is used to record oscillations of the wall of any superficial artery, for which purpose a funnel is placed on the examined vessel. Volumetric sphygmography records total vibrations of the vascular wall that are converted into vibrations of a portion of the body (usually an extremity). Volumetric sphygmogram is taken by placing a special cuff on the extremity.

Plethysmography is an instrument that measures or detects changes in volume within an organ or whole body.

Rheography is a method used to study the filling of a part of the body with blood by graphically recording the fluctuations in the resistance of that part of the body.

Myography measures the velocity and intensity of muscular contractions.

Phlebography is the recording of the venous pressure.

53.

A patient has developed paroxysmal ventricular tachycardia against the background of cardiac infarction. What anti-arrhythmic drug should be chosen to avoid lowering cardiac output?

Explanation

Lidocaine is a class Ib antiarrhythmic drug. It is an amide local anesthetic. Lidocaine decreases the duration of action potential by blocking the Na+ channel in depolarized cells. It also shortens the effective refractory period of purkinje fibers. Because of its rapid onset and short duration of action, the drug is particularly useful in treating arrhythmias when they arise in emergency situations such as open-heart surgery, digitalis intoxication, myocardial infarction. Lidocaine can be administered intravenously and intramuscularly.

54.

Electrocardiogram of a young man reveals deviation of his electrical axis of heart to the left. This phenomenon can be caused by:

Explanation

The position of the electrical axis changes with changes of the position of the heart in the chest. If the diaphragm is high (in hypersthenics) the electrical axis of the heart is more horizontal (to the left). If the diaphragm is low (in asthenic persons), the electrical axis is more vertical.

The hypersthenic is characterized by the relative predominance of the transverse over the longitudinal dimensions of the body (compared with the normosthenic constitution).

The asthenic constitution is characterized by a considerable predominance of the longitudinal over the transverse dimensions of the body by the dominance of the limbs over the trunk (i.e. long limbs).

Normosthenic constitution is characterized by a well proportioned make-up of the body and is intermediate between the asthenic and hypersthenic constitutions.

55. During a surgery for femoral hernia a surgeon operates within the boundaries of femoral trigone. What structure makes up its upper margin?

Explanation

krushkrok No184 (2014)krushkrok No184a (2014)

Boundaries of femoral triangle:

*Superiorly: inguinal ligament

*Medially: medial border of adductor longus muscle

*Laterally: medial border of the Sartorius muscle

*Roof: fascia lata

*Floor: adductor longus muscle, pectineus muscle and the iliopsoas muscle

56.

An obstetrician-gynecologist measures pelvis size of a pregnant woman. A caliper was used to measure the distance between the two iliac crests. What measurement of large pelvis was made?

Explanation

image

The greater pelvis (pelvis major) is bounded by the ala of the iliac bones on each side by the fifth lumbar vertebra and the iliolumbar ligaments in the back. In the front and above, the greater pelvis is wide open. The female greater pelvis has the following dimensions:

·        Interspinous distance (distantia interspinosa): the distance between the two anterior superior iliac spines (spina iliaca anterior superior) of 26cm.

·        Intercristal distance (distantia intercristalis or distantia cristarum): the maximum distance between the iliac crests (cristae iliacae) which constitute 29cm.

Moreover, obstetricians measure the intertrochanteric distance (distantia intertrochanterica) in order to estimate the dimensions of the pelvis. It sizes about 32cm.

57.

A patient has arterial hypertension. What long-acting drug from the group of calcium channel blockers should be prescribed?

Explanation

Calcium-channel blockers: verapamil, diltiazem, nifedipine, amlodipine, felodipine, nicardipine.

The dihydropyridines are calcium channel blockers with 2 generations (e.g. nifedipine, amlodipine, felodipine etc). all dihydropyridines have a much greater affinity for vascular calcium channels than for calcium channels in the heart. They are therefore, particularly attractive in treating hypertension.

The intracellular concentration of calcium plays an important role in maintaining the tone of smooth muscle and in the contraction of the myocardium. Calcium channel blockers block the inward movement of calcium by binding to L-type calcium channels in the heart and in smooth muscle of the coronary and peripheral arteriolar vasculature. This causes vascular smooth muscle to relax, dilating mainly arterioles. Calcium-channel blockers do not dilate veins.

58.

A patient has been diagnosed with URTI. Blood serum contains immunoglobulin M. What stage of infection is it?

Explanation

[ARVI – Acute respiratory viral infection; URTI - Upper respiratory tract infection] High IgM level usually indicate an acute or primary infection because, it is the first antibody to appear in response to initial exposure to an antigen. Secondary or reinfection show an increase in IgG. IgM antibodies appear early in the course of an infection, this makes it useful in the diagnosis of infectious diseases. Demonstrating IgM antibodies in a patient’s serum indicates recent infection or in a neonate’s serum, it indicates intrauterine infection.

59. A 43-year-old patient suffers from acute pancreatitis with disrupted common bile duct patency. What condition can develop in this case?

Explanation

Obturation (obstruction, to close) of bile duct – it can be:

* Intrahepatic – blockage of intrahepatic bile ducts

* Extrahepatic – blockage of common bile duct (ductus choledochus).

Findings:

* malabsorption: bile salts do not enter the Small Intestine; no emulsification of fat.

*light coloured stool: due to lack of urobilin (which leads to lack of stercobilin).

*Jaundice (posthepatic, mechanic, obstructive): increased conjugated Bilirubin.

* Steatorrhea

The findings are specific for obstruction of bile duct and bile acid deficiency.

60.

A patient has a tumor of the eyesocket tissues behind the eyeball. Disruption of accommodation and pupil constriction is observed. What anatomical structure is damaged?

Explanation

image

The Ciliary ganglion (ganglion ciliare) is a small group of postsynaptic parasympathetic nerve cell bodies associated with ophthalmic nerve (CNV1). The nasociliary nerve is associated with a small parasympathetic ganglion situated within the orbit laterally from the optic nerve. It is located between the optic nerve and the lateral rectus toward the posterior limit of the orbit. The ganglion receives nerve fibers from three sources:

·        Sensory fibers from CNV1 (ophthalmic branch of trigeminal nerve) via the sensory or nasociliary root of the ciliary ganglion.

·        Presynaptic parasympathetic fibers from CNIII (oculomotor) via the parasympathetic or oculomotor root of the ciliary ganglion.

·        Postsynaptic sympathetic fibers from the internal carotid plexus via the sympathetic root of the ciliary ganglion.

The postsynaptic fibers join the short ciliary nerves and proceed to the eyeball. The sensory fibers also join the short ciliary nerves to provide sensory nerve supply to the fibrous and the vascular layers of the eyeball.

61. Lymphocytes and other cells of our body synthesize universal antiviral agents as a response to viral invasion. Name these protein factors:

Explanation

Interferons are glycoprotein molecules. These cytokines are considered as antiviral agents. Interferons are of 3 types namely INF-α, INF-β and INF-γ. Alpha and beta interferons are induced by viruses, whereas gamma (T cell, immune) interferon is induced by antigens and is one of the effectors of cell-mediated immunity. Interferons are cytokines that inhibit the growth of certain cancer cells, bacteria and protozoa, but the focus here will be on their inhibitory effect on viral growth.

Interferon is a family of naturally occurring, inducible glycoproteins that interfere with the ability of viruses to infect cells. The antiviral mechanism is incompletely understood. It appears to involve the induction of host cell enzymes that inhibit viral RNA translation, ultimately leading to the degradation of viral mRNA and tRNA.

62.

A patient has undergone surgical removal of a cavitary liver lesion 2 cm in diameter. It was revealed that the cavity wall was formed by dense fibrous connective tissue; the cavity contained murky thick yellowish-green fluid with an unpleasant odor. Microscopically the fluid consisted mainly of polymorphonuclear leukocytes. What pathological process are these morphological changes typical for?

Explanation

Chronic abscess has internal pyogenic membrane; middle – granulation tissue; external – fibrous tissue membrane. Acute abscess basically has only a pyogenic membrane no fibrous tissue membrane. Phlegmon: diffuse purulent inflammation which occurs along muscular fibers, tendons, fascias etc. Empyema is a purulent inflammation of serous membranes (e.g. empyema of pleura). Both phlegmon and empyema does not involve the formation of cavity (“cavitary lesion”).

63.

A patient consulted a physician about chest pain, cough, fever. Roentgenography of lungs revealed eosinophilic infiltrates which were found to contain the larvae. What kind of helminthiasis are these presentations typical for?

Explanation

Ascaris lumbricoides (giant roundworm) is the causative agent of Ascariasis. Transmission: fecal-oral; eggs are visible in faeces under microscope. Some patients may have pulmonary symptoms or neurological disorders during migration of the larvae. A bolus of worms may obstruct the intestine; migrating larvae may cause pneumonitis and eosinophilia.

Ascaridiasis (Ascariasis) caused by Ascaris lumbricoides (giant roundworm). The major damage occurs during larval immigration rather than from the presence of the adult worm in the intestines. The principal sites of tissue reaction are the lungs, where inflammation with an eosinophilic exudates occurs in response to larval antigens. Ascaris pneumonia with fever, cough and eosinophilia can occur with a heavy larval burden.

64.

During appendectomy a patient had the a. appendicularis ligated. This vessel branches from the following artery:

Explanation

image

image

The ileocolic artery (arteria ileocolica) is a branch of superior mesenteric artery. The ileocolic artery arises from the right aspect of the main trunk. The artery descends rightwards and reaches the ileocecal angle to give off the branches. The arcades give off the branches to the caecum (the anterior and posterior caecal arteries); to the vermiform appendix (the appendicular artery) and to the terminal portion of the ileum (the ileal branches).

65. A 28-year-old patient undergoing treatment in a pulmonological department has been diagnosed with pulmonary emphysema caused by splitting of alveolar septum by tissular tripsin. The disease is caused by the congenital deficiency of the following protein:

Explanation

Emphysema: permanent enlargement of all or part of the respiratory unit. Causes include smoking cigarette (most common cause) and α1-antitrypsin (AAT) deficiency. α1-antitrypsin is a protease inhibitor; also referred to as α1-proteinase inhibitor because it inhibits a wide variety of proteases (including trypsin as the name implies).

66.

A patient, who has been suffering for a long time from intestine disbacteriosis, has increased hemorrhaging caused by disruption of posttranslational modification of blood-coagulation factors II, VII, IХ, and Х in the liver. What vitamin deficiency is the cause of this condition?

Explanation

image

Vitamin K is essential for the formation of various clotting factors in the liver, namely clotting factor II, VII, IX and X; Protein C and S.

Factor II – Prothrombin

Factor VII – Stable factor

Factor IX – Christmas factor

Factor X – Stuart-Prower factor

These factors undergo vitamin K-dependent post-translational modification, whereby a number of their glutamic acid residues are carboxylated to form ɣ-carboxyglutamic acid residues. The ɣ-carboxyglutamyl residues bind calcium ions which are essential for interaction between the coagulation factors and platelet membranes. Oral anticoagulants block epoxide reductase and creation of active form of vitamin K resulting in disturbances in prothrombin and proconvertin synthesis in liver.

Deficiency of vitamin K → ↓clotting factors → prolongation of coagulation time → hemorrhages.

67.

A 15-year-old patient has fasting plasma glucose level 4,8 mmol/l, one hour after glucose challenge it becomes 9,0 mmol/l, in 2 hours it is 7,0 mmol/l, in 3 hours it is 4,8 mmol/l. Such parameters are characteristic of:

Explanation

Diabetes mellitus (DM) refers to a group of common metabolic disorders that share the phenotype of hyperglycemia. Criteria for the diagnosis of DM:

·        Symptoms of diabetes plus random blood glucose concentration ≥11.1mmol (200mg/dL)

·        Fasting plasma glucose ≥7.0mmol/L (126mg/dL)

·        Glycated hemoglobin A1c >6.5%

·        Two-hour plasma glucose ≥11.1mmol/L (200mg/dL) during an oral glucose tolerance test.

NB: the question does not fulfill these criteria; therefore it is neither type I nor type II diabetes mellitus.

The diagnosis of diabetes can be made on the basis of individual’s response to the Oral glucose load, commonly referred to as Oral glucose tolerance test (OGTT).

The fasting plasma glucose level is <6.1mmol/L in healthy (normal) patients. On oral glucose load, the concentration increases and the peak value is reached in less than an hour which returns to normal by 2hrs.

In individuals with impaired glucose tolerance (which the question referred to as subclinical diabetes mellitus), the fasting as well as 2 hours plasma glucose levels are elevated. [Note: at the 2hr mark, it is still elevated]. These subjects slowly develop frank diabetes at an estimated rate of 2% per year. Dietary restriction and exercise are advocated for the treatment of impaired glucose tolerance.

68.

A 30-year-old man with diabetes mellitus type I was hospitalised. The patient is comatose. Laboratory tests revealed hyperglycemia and ketonemia. What metabolic disorder can be detected in this patient?

Explanation

During insulin deficiency, glucose cannot be utilized by the peripheral tissues for energy. So, a large amount of fat is broken down to release energy. It causes the formation of excess ketone bodies leading to acidosis. One more reason for acidosis is that the ketone bodies are excreted in combination with sodium ions through urine (ketonuria). Sodium exchanged H+, which diffuse from the renal tubules into ECF adding to acidosis.  In diabetic patients, the body produces too much acid & the kidneys are not removing them fast enough as they are produced which results in metabolic acidosis.

69.

A therapeutist has an appointment with a 40-year-old patient complaining of recurrent pain attacks in his hallux joints and their swelling. Urine analysis revealed its marked acidity and pink colour. What substances can cause such changes in the urine?

Explanation

krushkrok No21 (2014)

This is a classic description of gout. The end product of the purine nucleotides catabolism in humans and other primates is uric acid (urate) which is excreted in urine. Allopurinol and febuxostat inhibits Xanthine oxidase (XO). Hypoxanthine and Xanthine which is more soluble is excreted in urine. Purine nucleotides (adenine and guanine).  AMP – Adenosine monophosphate; GMP – Guanosine monophosphate

70.

A 47-year-old man developed intestinal colic against the background of essential hypertension. In this situation it would be most efficient to arrest the colic by administering drugs of the following group:

Explanation

Only myotropic antispasmodics (muscle relaxants) can effectively manage both intestinal colic and essential hypertension. Because sympathomimetics and adrenomimetics will exacerbate hypertension the more. Anticholinesterase agents and M-cholinomimetics will ↑acetylcholine effects in neuromuscular junction → ↑arteriolar smooth muscle tone → exacerbate hypertension. But the myotropic antispasmodics can ↓ muscle tone in intestinal smooth muscle (thereby relieving intestinal colic) and also ↓ arteriolar smooth muscle tone (thereby relieving essential hypertension).

71.

Microscopy of a female patient’s swabs made from vaginal secretion revealed gram-negative bean-shaped diplococci. What provisional diagnosis can be made?

Explanation

The genus Neisseria contains two important human pathogens: Neisseria meningitides and Neisseria gonorrhoeae. Neisseria meningitides mainly caused meningitis and meningococcemia. Neisseria gonorrhoeae causes gonorrhea, neonatal conjunctivitis (ophthalmia neonatorum) and pelvic inflammatory disease.

Neisseria are gram-negative cocci that resemble paired kidney beans. Gonococci cause disease only in humans. The organism is usually transmitted sexually; newborns can be infected during birth. Gonorrhea is usually symptomatic in men but often asymptomatic in women. Gonococci infect primarily the mucosal surfaces e.g. the urethra and vagina, but dissemination occurs.

Gonorrhea in men is characterized primarily by urethritis accompanied by dysuria and a purulent discharge. Epididymitis can occur. In women, infection is located primarily in the endocervix, causing a purulent vaginal discharge and intermenstrual bleeding (cervicitis).

72.

A 54-year-old woman was brought to a casualty department after a car accident. A traumatologist diagnosed her with multiple fractures of the lower extremities. What kind of embolism is most likely to develop in this case?

Explanation

Embolism is occlusion of a vessel by material travelling in the circulation.

* Fat/Adipose embolism: obstruction of arterioles and capillaries by fat globules constitutes fat embolism. It may occur following severe fracture trauma to bones, inflammation of bones and soft tissues, fatty liver

* Thromboembolism: a detached thrombus or part of thrombus which may arise in the arterial or venous circulation.

* Gas embolism: two main forms of gas embolism are air embolism and decompression sickness. Air embolism is usually due to accidental pumping of air into the venous circulation during intravenous (IV) injection or transfusion ( bubble – air escaped).

Tissue embolism: fragments of tissue.

73.

The organisms to be identified have a nucleus surrounded by a nuclear membrane. Genetic material is concentrated predominantly in the chromosomes that consist of DNA strands and protein molecules. These cells divide mitotically. Identify these organisms:

Explanation

The fundamental difference between eukaryotes and prokaryotes is that eukaryotes do have ‘true’ nuclei containing their DNA, whereas the genetic material in prokaryotes is not membrane-bound. Eukaryotes contain multiple linear chromosomes with large amount of noncoding and repetitive DNA.

74.

A 2-year-old boy is diagnosed with Down syndrome. What chromosomal changes may be the cause of this disease?

Explanation

krushkrok No149 (2014)

Chromosomal disorders: trisomy 21 (down’s syndrome); trisomy 18 (edward’s syndrome); trisomy 13 (patau’s syndrome); monosomy X (turner’s syndrome-XO); trisomy X (XXX);

normal female (XX); normal male (XY)

75.

After a road accident a victim has tachycardia, arterial blood pressure 130/90 mm Hg, tachypnoe, the skin is pale and dry, excitation of central nervous system is observed. What shock stage is the patient most likely in?

Explanation

Pirogov’s stages of shock

* Erectile phase: is characterized by strong motor agitation, sweating, tremor of skeletal muscles, staggering gait, frequent urination, transient increase in blood pressure, heart rate and breath rate increases, body temperature also. Painful impulses reach CNS.

* Torpid phase: decompensation in CNS leads to deep oppression. Patient is motionless, does not answer questions or answers very silently and with long time of delay, reflexes are lowered or absent.

76. As a result of a mechanical injury an over 10 cm long portion of a peripheral nerve was damaged. This caused an impairment of the upper limb activity. The patient was offered nerve transplantation. What glial cells will participate in regeneration and provide the trophism of the injured limb?

Explanation

Schwann cells are the major glial cells in peripheral nervous system (PNS). They provide myelination (insulation); also promote axonal regeneration. Derived from neural crest. Increase conduction velocity via salutatory conduction at the nodes of ranvier, where there is a high concentration of Na+ channels. They may be injured in Guillain-Barrė syndrome. Fibrous cells and protoplasmic cells are types of astrocytes forming blood-brain barrier, supporting network, mainataining the chemical environment and involved in recycling of neurotrtansmitters. Microglial cells are derived from monocytes – they engulf and destroy microorganisms and cellular debris. Ependymal cells form the epithelial lining of the ventricles of the brain and spinal cord.

77.

A patient has been given atropine sulfate for rapid relief of spastic colon symptoms. The use of this drug is contraindicated during the following disease:

Explanation

Glaucoma: increased intraocular eye pressure. Atropine sulfate (M-cholinoblocker) blocks the acetylcholine response of the ciliary muscle of the eye and of the circular smooth muscles of the iris, producing cycloplegia and mydriasis; it may increase intraocular pressure especially in patients with narrow-angle glaucoma. At low doses, it decreases cardiac rate (bradycardia). Arterial blood pressure is unaffected by atropine but at toxic levels, it will dilate the cutaneous vasculature. It is not effective in promoting healing of peptic ulcer. It can also relieve bronchial asthma. It is contraindicated because its going to further increase intraocular pressure.

78.

As an example of specific human parasites one can name Plasmodium falciparum, human pinworm and some others. The source of parasite invasion in these cases is always a human. Such specific human parasites cause the diseases that are called:

Explanation

Anthroponoses: diseases transmissible from human to human. Zoonoses: diseases transmissible from living animals to humans; formerly called anthropozoonoses. And diseases transmissible from humans to animals are called zooanthroponoses.

79.  In the course of an experiment there has been increase in nerve conduction velocity. This may be caused by increase in concentration of the following ions that are present in the solution around the cell:

Explanation

Nerve signals are transmitted by action potential (AP), which are rapid changes in the membrane potential that spread rapidly along the nerve fiber membrane. During the depolarization stage of an AP, the membrane suddenly becomes permeable to sodium ion (Na+), allowing tremendous number of positively charged Na+ to diffuse to the interior of the axon. A major function of voltage-gated calcium ion (Ca2+) channels is to contribute to the  depolarizing phase on the action potential in some cells. Although the gating of calcium channel is slow (slow channels), in contrast to the fast sodium channels. Therefore, sodium channels play a key role in initiation and conduction of action potentials.

80.

An HIV-positive patient’s cause of death is acute pulmonary insufficiency resulting from pneumonia. Pathohistological investigation of lungs has revealed interstitial pneumonia, alveolocyte desquamation and methamorphoses: alveolocyte enlargement, large intranuclear inclusions surrounded by lightly-coloured areas. Transformed cells resemble owl’s eye. Name the causative agent of pneumonia:

Explanation

Cytomegalovirus (CMV) cause congenital infection, mononucleosis, pneumonia, retinitis. Infected cells have characteristic “owl-eye” inclusions. Transmitted congenitally and by transfusion, sexual contact, saliva, urine, transplant. Interstitial pneumonia is mainly caused by viruses or mycoplasma. CMV is responsible for the most common viral opportunistic infection in persons with HIV infection.

81.

A 65-year-old man suddenly lost vision in one eye due to the retinal detachment. The patient underwent enucleation. Histological examination of the removed eye retina and choroid revealed clusters of atypical cells with marked polymorphism of cells and nuclei, with a moderate number of mitoses including the pathological ones. The cell cytoplasm and intercellular medium contained brown pigment resulting in positive DOPA reaction. Perls’ reaction was negative. What is the most likely diagnosis?

Explanation

Melanoma is strongly linked to acquired mutations caused by exposure to ultraviolet (UV) radiation in sunlight. Melanomas show striking variations in colour, appearing in shades of black, brown, red, dark blue and gray. The borders of melanomas are irregular and often notched, unlike the smooth, round and uniform borders of melanocytic nevi. The positive DOPA reaction indicates the presence of melanocytes. Melanoma arises in the skin, meninges, uvea of the eye.

82. During determining the blood group according to the AB0 system with salt solutions of monoclonal antibodies agglutination did not occur with any of the solutions. What blood group is it?

Explanation

image

Blood group O(I): no antigens, therefore no agglutination.

Blood group A(II): A antigen, agglutinate with blood group B(III) and O(I).

Blood group B(III): B antigen, agglutinate with blood group A(II) and O(I).

Blood group AB(IV): A and B antigen, agglutinate with all blood groups. No antibody.

Blood group name is determined by the antigen present on RBC, but the patient has an opposite antibody. So whenever, the antibody corresponds to the antigen, there is agglutination. Since O does not have any antigen, no agglutination can occur.

83.

One of the factors that cause obesity is inhibition of fatty acids oxidation due to:

Explanation

image

The major pathway for catabolism of saturated fatty acids is a mitochondrial pathway called β-oxidation. After a long-chain fatty acid (LCFA) enters a cell, it is converted in the cytosol to its Co-A derivative. Because β-oxidation occurs in the mitochondrial matrix, the fatty acid must be transported across inner mitochondrial membrane which is impermeable to Co-A. therefore, a specialized carrier transports the long chain acyl group from the cytosol into the mitochondrial matrix. This carrier is carnitine and this rate-limiting transport process is called the carnitine shuttle. Since carnitine helps the mitochondria utilize energy, it plays a critical role in reducing the occurrence and impact of obesity. In addition to helping the mitochondria burn fat as energy, carnitine is also vital for removing waste products from mitochondria. Obesity and aging contribute to low carnitine levels, which compromises mitochondrial performance and increases insulin resistance, promoting further obesity and carnitine reduction.

84.

During ventricular systole the cardiac muscle does not respond to additional stimulation because it is in the phase of:

Explanation

krushkrok No145 (2013)

Absolute refractory period is the period during which the muscle does not show any response at all, whatever may be the strength of the stimulus. It is because the depolarization occurs during this period. So, a second depolarization is not possible in cardiac muscle, absolute refractory period extends throughout systole (contraction) period – 0.27sec. And, relative refractory period extends during first half of diastole (relaxation) period – about 0.26sec. Relative refractory period is the period during which the muscle shows some response if the strength of stimulus is increased to maximum.

85.

A patient is diagnosed with acute morphine hydrochloride intoxication. Prescribe the oxidizing agent for gastric lavage:

Explanation

Potassium permanganate: KMnO4 →MnO2 + O2. MnO2 has astringent action. O2  oxidizes proteins in microbes, deodorizing action, antidote action. 0.1% - 100ml for gastric lavage. Chloramines and chlorhexidine bigluconate – nonorganic antiseptic and disinfectant  → chlorine containing (halogens) for treatment of wounds and processing of surgeon’s hands. Sulfocamphocainum – respiratory stimulant.

86.

During cell division DNA replication occurs after a signal is received from the cytoplasm, then a certain portion of the DNA helix unwinds and splits into two individual strains. What enzyme facilitates this process?

Explanation

image

*Helicase unwinds DNA template at the replication fork.

*DNA-dependent RNA polymerase (primase) makes an RNA primer on which DNA polymerase III can initiate replication.

*Ligase seals fragments of newly synthesized DNA fragments (i.e. joins okazaki fragments).

*DNA polymerase III elongates leading strand and lagging strand. DNA polymerase I degrades RNA primer, replaces it with DNA.

87.

When treating a patient with chronic cardiac failure a doctor detected bradycardia and deterioration of the patient’s general state. Such condition is caused by cumulative effect of a drug. Which drug of those listed below has cumulative action?

Explanation

The symptoms describe cardiac insufficiency. Cardiac glycoside is the most appropriate in this case. Cardiac glycosides are glycosidic drugs of plant origin having cardiac inotropic property, increase myocardial contractility and output. Corglyconum is short acting for acute cardiac insufficiency and administered intravenously.

FullSizeRender (32)

Digitoxin is long-acting but basically administered orally and used mainly for chronic heart failure (also digoxin).

88. A doctor was addressed by a 30-year-old man. There is a probability of the patient being HIV-positive. To clarify the diagnosis the doctor proposed to perform polymerase chain reaction. The basic process in this kind of investigation is:

Explanation

Polymerase chain reaction (PCR) is a molecular biology laboratory procedure used t amplify a desired fragment of DNA. Useful as a diagnostic tool (e.g. HIV, herpes encephalitis).

Nucleic acid-based (DNA or RNA based) tests are highly specific, quite sensitive (especially the amplification tests) and mush faster than culturing the organism. Nucleic acid amplification tests utilize the PCR or other amplifying processes to increase the number of specific DNA or RNA molecules so the sensitivity of the test is significantly higher than that of unamplified tests. Steps in PCR includes: denaturation; annealing and elongation. These steps are repeated multiple times for DNA sequence amplification.

89. Due to the use of poor-quality measles vaccine for preventive vaccination, a 1-year-old child developed an autoimmune renal injury. The urine was found to contain macromolecular proteins. What process of urine formation was disturbed?

Explanation

When blood passes through glomerular capillaries, the plasma is filtered. All substances of plasma are filtered except the plasma proteins and plasma cells, due to their large molecular size which is larger than the slit pores present in the endothelium of capillaries. Glomerular capillary membrane, basement membrane and visceral layer of bowman capsule form the filtration membrane through which glomerular filtration occurs. So, if protein and erythrocytes are increased or present in urine, it has to do with increase in renal filter permeability.

90.

A 26-year-old female patient with bronchitis has been administered a broad spectrum antibiotic as a causal treatment drug. Specify this drug:

Explanation

The tetracyclines (minocycline, doxycycline) are broad spectrum bacteriostatic antibiotics.

Mechanism of action: entry of these drugs into susceptible organisms is mediated both by passive diffusion and by an energy-dependent transport protein mechanism. The drug binds reversibly to the 30S subunit of the bacterial ribosome, thereby blocking access of the amino acyl-tRNA to the mRNA-ribosome complex at the acceptor site.

Taking these drugs concomitantly with diary foods in the diet decreases absorption due to the formation of non-absorbable chelates of the tetracyclines with calcium ions. Non-absorbable chelates are also formed with other divalent and trivalent cations (e.g. those found in magnesium and aluminum antacids and in Iron preparations).

Adverse effects:

·         Discolouration and hypoplasia of the teeth in growing children

·         Phototoxicity: such as sunburn

·         Dysbacteriosis

·         Gastric discomfort

So far, Doxycycline has been the only tetracycline tested in Krok.

91.

During blood transfusion a patient has developed intravascular erythrocyte hemolysis. What kind of hypersensitivity does the patient have?

Explanation

* Type I: Allergic and atopic disorders (e.g. rhinitis, hay fever, asthma); Anaphylaxis. Presentation – immediate, anaphylactic, atopic

* Type II: Acute hemolytic transfusion reactions, autoimmune hemolytic anemia, Erythroblastosis fetalis, Goodpasture syndrome, Graves disease, Myasthenia gravis. Disease tends to be specific to tissue or site where antigen is found.

* Type III: Arthus reaction; SLE; serum sickness. Can be associated with vasculitis and systemic manifestations.

* Type IV: Contact dermatitis, Graft-versus-host disease, multiple sclerosis. Response is delayed and does not involve antibodies (vs. Type I, II, III)

92.

In the course of an experiment thalamocortical tracts of an animal were cut. What type of sensory perception remained intact?

Explanation

Olfactory receptors (1st order neuron) → Mitral cells (2nd order neuron) → Olfactory tract → Olfactory trigone, anterior perforated substance, septum pellucidum (3rd order neuron) → Uncus of parahippocampal gyrus.

The Olfactroy analyzer is one of the oldest ones so it features several fibers that take the shortest route to the Olfactory cortex i.e. do not relay within the thalamus.

Gustatory: through its 2nd order neuron from medial lemniscus → thalamus.

Visual: through its subcortical visual centers → pulvinar of thalamus

Tactile (skin analyzer); Auditory: through lateral lemniscus of inferior colliculi of tectal plate and medial geniculate bodies of metathalamus.

93.

A 4-year-old child presents with general weakness, sore throat and deglutitive problem. After his examination a doctor suspected diphtheria and sent the material to a bacteriological laboratory. To determine the diphtheria causative agent the material should be inoculated into the following differential diagnostic medium:

Explanation

Diphtheria bacteria (Corynebacterium diphtheria) is Gram positive, pleomorphic, often club-shaped rods and are arranged in palisades or in V (at an angle) or L-shaped formations. Media used for isolation are Tellurite agar & Lὄffler medium. Lὄffler nutrient medium consists of coagulated serum & nutrient broth. Selective indicator medium containing tellurite are used in selective culturing. K tellurite is used to inhibit the accompanying flora.

94.

A patient was prescribed loratadine to treat allergic cheilitis. What is the mechanism of action of this drug?

Explanation

  Loratidine : Antihistamine (antiallergic) block H1-histamine receptors as well as adrenergic, cholinergic & serotonin-binding recpetors. They do not influence the formation or release of histamine, but rather they competitively block the receptor-mediated response of target tissue. Benadryl (Diphenhydramine, Dimedrol): Antihistamine (1st generation) – H1 receptor blocker. Has sedative, hypnotic and ganglioblocking effects. Loratadine (2nd generation) – also  an antihistamine but has a weak sedative, hypnotic effect.                                                                                          
95.

During introduction of local anesthesia a patient has gone into anaphylactic shock. What drug must be administered to the patient?

Explanation

* Epinephrine/Adrenaline hydrochloride: is a α-β-adrenergic agonists (adrenomimetics), direct acting. Uses: Anaphylactic shock, hypoglycemic coma…

* Diazepam: Tranquilizer (Benzodiazepine). Uses: anxiety, status epilepticus, night terrors.

* Atropine sulphate: M-cholinoblocker. Uses: eye traumas, GI diseases with ↑acidity.

* Propanolol: β-blocker. Uses: hypertension, Ischemic heart disease.

* Nitroglycerine: Organic nitrates. Uses: angina attack

96.

A patient has been diagnosed with influenza. His condition drastically worsened after taking antipyretic drugs. He is unconscious, AP is 80/50 mm Hg, Ps is 140/m, body temperature dropped down to 35,8oC . What complication developed in this patient?

Explanation

His condition became worse after taking antipyretic drug. Antipyretic drugs block prostaglandin synthesis by inhibiting cyclooxygenase (COX) enzyme at the thermoregulating centers in the hypothalamus and at peripheral target sites. In case of toxicity of this drug, it can result in coma followed by cardiovascular collapse and respiratory arrest due to its CNS depressant activity. Consciousness is confused; arterial blood pressure is low and pulse rate is very high – these are clear signs of collapse.

97.

An alcoholic has alcoholic psychosis with evident psychomotor agitation. What neuroleptic drug should be administered for emergency aid?

Explanation

·        Aminazine – neuroleptic (it is better than reserpine for emergency care. It has also proved to be useful in control of intracted hiccup).

·        Diazepam – tranquilizer

·        Sodium bromide – sedative

·        Reserpine – neuroleptic

·        Halothane – general anesthesia

98. A patient had a trauma that caused dysfunction of motor centres regulating activity of head muscles. In what parts of cerebral cortex can the respective centre normally be located?

Explanation

image

IMG_9921

Primary motor area extends throughout the precentral gyrus and the adjoining lip of central sulcus. Areas 4 and 4S are present here. Muscles of various parts of the body are represented in area 4 in an inverted way from medial to lateral surface. Lower parts of the body are represented in the lateral (upper) surface and upper parts of the body are represented in the lateral (lower) surface. Order of representation from medial (superior, upper) to lateral (inferior, lower) surface: toes, ankle, knee, hip, trunk, shoulder, arm, elbow, wrist, hand, fingers and face. Area 4 is concerned with contraction of discrete muscles.

99.

During intravenous saline transfusion a patient’s condition deteriorated drastically, and the patient died from asphyxiation. Autopsy revealed acute venous congestion of internal organs with sharp right heart dilatation. When the right ventricle was punctured underwater, the bubbles escaped. What pathological process occurred in the patient?

Explanation

Embolism is occlusion of a vessel by material traveling in the circulation.

* Gas embolism: two main forms of gas embolism are air embolism and decompression sickness. Air embolism is usually due to accidental pumping of air into the venous circulation during intravenous (IV) injection or transfusion ( bubble – air escaped).

* Fat/Adipose embolism: obstruction of arterioles and capillaries by fat globules constitutes fat embolism. It may occur following severe fracture trauma to bones, inflammation of bones and soft tissues, fatty liver

* Thromboembolism: a detached thrombus or part of thrombus which may arise in the arterial or venous circulation.

Tissue embolism: fragments of tissue.

100. Diseases of respiratory system and circulatory disorders impair the transport of oxygen, thus causing hypoxia. Under these conditions the energy metabolism is carried out by anaerobic glycolysis. As a result,  the following substance is generated and accumulated in blood:

Explanation

Lactate, formed by the action of lactate dehydrogenase (converting pyruvate to lactate) is the final product of anaerobic glycolysis in eukaryotic cells. In organs or cells that are poorly vascularized and/or lack mitochondria, formation of lactate is the major fate of pyruvate as seen in lens, cornea of the eye, kidney medulla, testes, leukocytes and red blood cells.

Aerobic glycolysis progresses to citric acid cycle from pyruvate. The cycle occurs totally in the mitochondria.

101.

Examination of a 6-month-old child revealed a delay in closure of the occipital fontanelle. When should it normally close?

Explanation

font....

Fontanelles are large fibrous areas where several sutures meet; often called “soft spots” on an infant’s head. The two largest fontanelles are the anterior and posterior fontanelles on the superior surface of the neurocranium. The fontanelle in the back of the head (posterior fontanelle) usually closes by the time an infant is 2-3 months old. The fontanelle at the top of the head (anterior fontanelle) usually closes between 7-18months. In adults the remnants of the anterior fontanelle is the bregma and the posterior fontanelle is the lambda.

102. Sex chromosomes of a woman didn’t separate and move to the opposite poles of a cell during gametogenesis (meiosis). The ovum was impregnated with a normal spermatozoon. Which chromosomal disease can be found in her child?

Explanation

image

krushkrok No105 (2014)In meiosis, two members of a pair of homologous chromosomes normally separate during the first meiotic division, so that each daughter cell receives one member of each pair. Sometimes, however, separation does not occur (nondisjunction) and both members of a pair move into one cell. As a result of nondisjunction of the chromosomes, one cell receives 24 chromosomes and the other receives 22 instead of the normal 23. When at fertilization, a gamete having 23 chromosomes (spermatozoon) fuses with a gamete having 24 or 22 chromosomes, the result is an individual with either 47 chromosomes (47 XXX – trisomy X) or 45 chromosomes (45 XO – monosomy X, Turner’s syndrome).

In women, the incidence of chromosomal abnormalities including nondisjunction, increases with age especially at 35years and older.

103.

In a cat with decerebrate rigidity the muscle tone is to be decreased. This can be achieved by:

Explanation

Decerebrate rigidity indicates brain stem damage, specifically damage below the level of the red nucleus. Vestibular nerve has 4 nuclei – superior(Bechterew’s), inferior(Roller), lateral(Deiter’s) and medial(Schwalbe’s). In the lateral vestibular nucleus, some fibers from this nucleus pass into the cerebellum, which is important in regulating muscle tone. It has been shown experimentally that destruction of this nucleus leads to loss of tone almost completely.

104.

Autopsy of a 5-year-old child revealed in the area of vermis of cerebellum a soft greyish-pink node 2 cm in diameter with blurred margins and areas of haemorrhage. Histologically this tumour consisted of atypical monomorphous small round cells with large polymorphous nuclei. What tumour is it?

Explanation

Medulloblastoma is a tumor made by immature cells, medulloblasts; therefore it is highly malignant. It is localized in the vermis of cerebellum. Macroscopically, it is pinkish-gray. Microscopically, medulloblastoma consists of homogenous small cells with dark round or oval nucleus and poorly seen rim of cytoplasm. The cells are located close to each other. Rosette is typical. Mitoses are numerous. Vessels are not numerous. Metastasis spread through the liquor routs.

105.

Surface with an intact toad on it was inclined to the right. Tone of extensor muscles became reflectory higher due to the activation of the following receptors:

Explanation

image     

The receptor areas of the vestibular labyrinth are represented with the following structures:

·        Macula of Utricle

·        Macula of Saccule

·        Ampullary crests

The first two detect linear motion, while the ampullary crest detect angular motion. The vestibulocochlear nerve supply these receptors. Both maculae consist of the sensory hair cells covered with jelly-like susbstance. The substance contains the crystals of calcium carbonate called otoliths (vestibular otolith). The utricle and saccule detect linear movement, also contributing to balance. The ampullary crests reside within each membranous ampulla. They also comprise the sensory hair cells covered with the same jelly-like substance called the ampullary cupula. The ampullae is the sensory organ in the semicircular canal that sense angular (rotational) acceleration of the head, thereby regulating balance.

106. A patient with high-titer antinuclear antibodies died from progressing renal impairment. Autopsy revealed mesangioproliferative glomerulonephritis and a bacterial polypous endocarditis. Periarterial bulbar sclerosis was detected in spleen and productive proliferative vasculitis in skin. What is the most likely diagnosis?

Explanation

Systemic lupus erythematous (SLE, Libman-sacks disease) is the classic prototype of the multisystem disease of autoimmune origin, characterized by a bewildering array of autoantibodies, particularly antinuclear antibodies. It is characterized principally by injury to the skin, joints, kidney, and serosal membranes. Antinuclear antibody is directed against several nuclear antigens and can be grouped into 4 categories:

·        Antibodies to DNA

·        Antibodies to histones

·        Antibodies to nonhistone proteins bound to RNA

·        Antibodies to nuclear antigens.

SLE is a type III hypersensitivity reaction with formation of immune complexes. It can cause diffuse proliferative glomerulonephritis seen under the light microscope as “wire loop” of capillaries and granular under the immunofluorescence microscopy.

107.

When studying the signs of pulmonary ventilation, reduction of forced expiratory volume has been detected. What is the likely cause of this phenomenon?

Explanation

Obstructive respiratory disease is the abnormal respiratory condition characterized by difficulty in expiration. E.g asthma, chronic bronchitis, emphysema, cystic fibrosis. Forced expiratory volume (FEV) is the volume of air, which can be expired forcefully in a given unit of time. It is very much decreased in obstructive diseases like asthma and emphysema.                                           Restrictive respiratory disease is the abnormal respiratory condition characterized by difficulty in inspiration. E.g poliomyelitis, myasthenia gravis, paralysis of diaphragm, spiral cord diseases, pleural effusion, fibrosis. (lung fibrosis-pneumofibrosis)
108.

A specimen of a parenchymal organ shows poorly delineated hexagonal lobules surrounding a central vein, and the interlobular connective tissue contains embedded triads (an artery, a vein and an excretory duct). What organ is it?

Explanation

krushkrok No118a (2013)

There are 3 ways to describe the structure of the liver in terms of a functional unit: the classic lobule; portal lobule and the liver acinus. The classic lobule is the traditional way to describe the organization of the liver parenchyma. The classic hepatic lobule is a roughly hexagonal (6-sided) mass of tissue. At the center of the lobule is a relatively large venule, the terminal hepatic venule (central vein), into which the sinusoids drain. At the angles of the hexagon are the portal areas (portal canals), loose stromal connective tissue characterized by the presence of the portal triad. The portal triad is composed of the branches of the hepatic artery, portal vein and the bile duct.

image

109.

A doctor examined a patient, studied the blood analyses, and reached a conclusion, that peripheral immunogenesis organs are affected. What organs are the most likely to be affected?

Explanation

Organs of the immune system are divided into two groups:

·         Central (primary): thymus and bone marrow

·         Peripheral (secondary): lymph nodes, spleen, lymphatic tissue of GIT, tonsils

The tonsils begin to form on the 22nd week of a fetus life, but develop until the puberty period. They consist basically of the reticular tissue. The lymphoid tissue represents follicles by itself. There are T- and B-lymphocytes in the tonsils. The tonsils form a ring of lymphatic tissue at the entrance of the oropharynx (e.g. pharyngeal, palatine, lingual tonsils).

110. A doctor asked a patient to make a deep exhalation after a normal inhalation. What muscles contract during such exhalation?

Explanation

All muscles that elevate the rib cage are muscles of inspiration and those that depress the rib cage are muscles of expiration.

Muscles of inspiration:

·        Sternocleidomastoid: lift upward on the sternum.

·        Anterior serrati: lift many of the ribs.

·        Scalene: lift the first two ribs.

Muscles of expiration: Abdominal recti – pull down the lower ribs and other abdominal muscles also compress the abdominal contents upwards against the diaphragm and internal intercostals.

Normal respiration is accomplished by the movement of the diaphragm only.

111. A 4-year-old child with hereditary renal lesion has signs of rickets; vitamin D concentration in blood is normal. What is the most probable cause of rickets development?

Explanation

image

Rickets results from insufficiency of vitamin D (calcitriol). Calcitriol is the active form of vitamin D. calcitriol production is dependent on the kidney’s 1-α-hydroxylase which converts 25-OHD3 to 1,25-(OH)2D3 (calcitriol). So in cases of renal lesion, there is lack of 1-α-hydroxylase which leads to a deficiency of calcitriol or impaired synthesis of calcitriol.

112.

Parents of a 5-year-old boy report him to have frequent colds that develop into pneumonias, presence of purulent rashes on the skin. Laboratory tests have revealed the following: absence of immunoglobulins of any type, and naked cells are absent from the lymph nodes punctate. What kind of immune disorder is it?

Explanation

X-linked (Bruton) agammaglobulinemia: defect in Bruton’s tyrosine kinase (BTK), a tyrosine kinase gene → no B cell maturation. X-linked recessive (↑in boys). Findings: absent B cells in peripheral blood, ↓immunoglobulin of all classes. Absent/scanty lymph nodes and tonsils.

113. Examination of a 42-year-old patient revealed a tumour of adenohypophysis. Objectively: the patient’s weight is 117 kg, he has moon-like hyperemic face, red-blue striae of skin distension on his belly. Osteoporosis and muscle dystrophy are present. AP is 210/140 mm Hg. What is the most probable diagnosis?

Explanation

krushkrok No159 (2012)krushkrok No159a (2012)

Cushing Syndrome: Etiology

*INCREASE Cortisol due to a variety of causes (Glucocorticoids).

*Exogenous corticosteroids:  result in DECREASE ACTH (MCC).

*Primary adrenal adenoma, hyperplasia or carcinoma (Cushing’s Syndrome).

*ACTH-secreting pituitary adenoma (Cushing Disease).

Findings: Hypertension, Weight Gain, Moon Facies, Truncal Obesity, Buffalo Hump, Skin Changes (thinning striae), Osteoporosis, Hyperglycemia (Insulin resistance), Amenorrhea, Immunosuppression.

114.

2 days after labour a woman developed shock along with DIC syndrome that caused her death. Autopsy revealed purulent endomyometritis, regional purulent lymphangitis, lymphadenitis and purulent thrombophlebitis. There were also dystrophic alterations and interstitial inflammation of parenchymal organs. What is the most likely diagnosis?

Explanation

Septicemia is a form of sepsis. Hemorrhagic syndrome is well pronounced (petechial rash, hemorrhages to the serous and mucus membranes and internal organs, DIC). Hyperplasia of lymphoid and hemopoietic system is typical. Proliferation of lymphoid and reticular cells as well as accumulation of mature and immature blood cells are found in the spleen and lymph nodes. Interstitial inflammation develops  in the parenchymal organs (heart, liver, kidneys). Septicemia can lead to septic shock. DIC – disseminated intravascular coagulation

115.

In case of alkaptonuria, homogentisic acid is excreted in urine in large amounts. The development of this disease is associated with metabolic disorder of the following amino acid:

Explanation

image

Ochronosis (Alkaptonuria): congenital deficiency of homogentisate oxidase (homogentisic acid oxidase) in the degradative pathway of tyrosine to Fumarate → pigment-forming homogentisic acid accumulates (homogentisuria) in tissues. Autosomal recessive. Usually benign. Urine turns black on prolonged exposure to air. May have debilitating arthralgias (homogentisic acid toxic to cartilage).

Phenylalanine → Tyrosine →→→ Homogentisic acid → Maleylacetoacetic acid

116.

When blood circulation in the damaged tissue is restored, lactate accumulation stops and glucose consumption decelerates. These metabolic changes are caused by activation of the following process:

Explanation

Lactate, formed by the action of lactate dehydrogenase (converting pyruvate to lactate) is the final product of anaerobic glycolysis in eukaryotic cells. In organs or cells that are poorly vascularized and/or lack mitochondria, formation of lactate is the major fate of pyruvate as seen in lens, cornea of the eye, kidney medulla, testes, leukocytes and red blood cells.

Aerobic glycolysis progresses to citric acid cycle from pyruvate, especially when there is blood supply to make the necessary oxygen available for the process. The cycle occurs totally in the mitochondria.

117.

Experimental stimulation of the sympathetic nerve branches that innervate the heart caused an increase in the force of heart contractions because the membrane of typical cardiomyocytes permitted an increase in:

Explanation

IMG_9907 IMG_9903 When a muscle is excited (stimulated) by the impulses passing through neuromuscular junction, action potential is generated which spreads over sarcolemma (plasma membrane of muscles). When the action potential reaches the cisternae of ‘L’ tubules, Ca2+ stored in the cisternae are released into the sarcoplasm (cytoplasm of muscles). The Ca2+ moves towards the actin filaments to produce the contraction. Therefore, it is Ca2+ entry into the sarcoplasm.
118.

According to the results of glucose tolerance test a patient has no disorder of carbohydrate tolerance. Despite that glucose is detected in the patients’ urine (5 mmol/l). The patient has been diagnosed with renal diabetes. What renal changes cause glucosuria in this case?

Explanation

Food substances like glucose, amino acids, acetoacetate ions and vitamins are completely reabsorbed from renal tubules and do not appear in urine under normal conditions. These substances can appear in urine, only if their concentration in plasma is abnormally high (for glucose above 10mmol/L – renal threshold) or in renal diseases when reabsorption is affected. Since glucose is 5mmol/L, then it has to be a problem with decreased reabsorption of glucose.
119.

Alveolar space of acinus was invaded by bacteria that interacted with the surfactant. This led to the activation of the cells that are localized in the alveolar walls and on the surface. Name these cells:

Explanation

Alveolar macrophages remove inhaled particulate matter from the air spaces and red blood cells (RBCs) from the septum of alveoli. They are unusual in that they function both in the connective tissue of the septum (alveolar wall) and in the air space of the alveolus (surface of alveolar cells). In air spaces they scavenge the surface to remove inhaled particulate matter (e.g. dust, pollen, pathogens), thus giving them one of their alternate names – Dust cells. They also phagocytose infectious organisms such as Mycobacterium tuberculosis. Other macrophages remain in the septal connective tissue, where, filled with accumulated phagocytized material, they may remain for much of an individual’s life. Thus, at autopsy, the lungs of urban dwellers as well as smokers will usually show many alveolar and septal macrophages filled with carbon particles, anthracotic pigment and birefringent needle-like particles of silica.

Type I alveolar cells (type I pneumocytes): the junctions formed between this cells form an effective barrier between the air space and the components of the septal wall. They are not capable of cell division. They make up 95% of the surface of the alveoli.

Type II alveolar cells: secretory cells; have lamellar bodies; progenitor cells for type I alveolar cells.

Clara cells: non-ciliated; low columnar/cuboidal cell with secretory granules. They secrete components of surfactant, degrade toxins and act as reserve cells.

Endothelial cells line blood vessels.

120.

When playing a child received a hit to the presternum region. As a result of this trauma an organ located behind the presternum was damaged. Name this organ:

Explanation

The thymus resides in the anterior mediastinum posterior to the manubrium of sternum reaching the IV rib. Mediastinal pleurae neighbor the thymus laterally, the pericardium, arch of aorta with associated branches and inferior vena cava posteriorly. Topographically, the thymus is closer to the sternum than the heart. So it will be affected first by a blow to the sternum because it lies outside of the pericardial cavity (anterior to the fibrous pericardium).

121.

A child suffers from dry cough. What non-narcotic antitussive drug will relieve the patient’s condition?

Explanation

Narcotic – Opoid. Tussive – Cough.

Glaucine hydrochloride is the only antitussive centrally acting but non-opoid (non-narcotic).

Codeine phosphate and Morphine hydrochloride - antitussive centrally acting but opoid.

122.

A patient complains of acute pain attacks in the right lumbar region. During examination a nephrolithic obturation of the right ureter in the region between its abdominal and pelvic segments has been detected. What anatomical boundary exists between those two segments?

Explanation

krushkrok No77 (2014)

Linea terminalis = pectineal line (pubis) + Arcuate line + sacral promontory + superior margin of pubic symphysis. Boundary between the abdominal and pelvic cavity.

Linea semilunaris found on the lateral margin of rectus abdominis.

Linea arcuata: the region on the posterior layer, where aponeuroses end and continue into the anterior layer. Part of linea terminalis anteriorly.

123.

A patient has insufficient blood supply to the kidneys, which has caused the development of pressor effect due to constriction of arterial resistance vessels. This condition results from the vessels being strongly affected by the following substance:

Explanation

FullSizeRender (12)

↓blood supply → activates renin. Renin catalyzes the conversion of angiotensinogen to angiotensin I. Angiotensin converting enzyme (ACE) converts angiotensin I → angiotensin II. Angiotensin II causes:

- vasoconstriction → ↑BP

- vasopressin (ADH) → ↑H2O reabsorption → ↑plasma volume

- aldosterone → ↑Na+ and H2O reabsorption → ↑plasma volume → ↑BP

124.

In a village a case of anthrax has been registered. Medical services began epidemiologically indicated specific prophylaxis of population against anthrax. What preparation was used for this purpose?

Explanation

Live spore vaccine (STI) is used for vaccination against anthrax. STI live vaccine
125.

A 66-year-old woman had intravenous injection of magnesium sulfate solution to stop hypertensive crisis. However her arterial pressure did not decrease and after repeated introduction of the same preparation she developed sluggishness, slow response to stimuli; the patient is unconsciousness and her respiration is inhibited. What preparation is antagonist of magnesium sulfate and can remove the symptoms of its overdose?

Explanation

Magnesium sulfate reduces striated muscle contractions and blocks peripheral neuromuscular transmission by reducing acetylcholine release at the neuromuscular junction. Magnesium is a unique calcium antagonist as it can act on most types of calcium channels in vascular smooth muscle and as such would be expected to decrease intracellular calcium. One major effect of decreased intracellular calcium would be inactivation of calmodulin-dependent myosin light chain kinase activity and decreased contraction, causing arterial relaxation that may subsequently lower peripheral and cerebral vascular resistance, relieve vasospasm and decrease arterial blood pressure. Therefore, it is recommended that intravenous calcium gluconate or calcium chloride should be available as an antagonist of magnesium to increase intracellular calcium level.

126.

A patient working at a pig farm complains of paroxysmal abdominal pain, liquid feces with mucus and blood, headache, weakness, fever. Examination of large intestine revealed ulcers from 1 mm up to several cm in diameter, feces contained oval unicellular organisms with cilia. What disease can be suspected?

Explanation

IMG_9915   

 Balantidium coli causes balantidiasis. It is the only ciliated protozoan that causes human disease i.e. diarrhea. Domestic animals, especially pigs are the main reservoir for the organism and humans are infected after ingesting the cysts in food or water contaminated with animal or human faeces. Diagnosis is made by finding large ciliated trophozoites or large cysts with a characteristic V-shaped nucleus in the stool. The trophozoites excyst in the small intestine, travel to the colon (large intestine) and by burrowing into the wall cause an ulcer similar to that of Entamoeba histolytica.

127. An unconscious patient was delivered by ambulance to the hospital. On objective examination the patient was found to present no reflexes, periodical convulsions, irregular breathing. After laboratory examination the patient was diagnosed with hepatic coma. Disorders of the central nervous system develop due to accumulation of the following metabolite:

Explanation

Substances absorbed into the bloodstream from the intestine pass through the liver, where toxins are normally removed. Many of these toxins (such as ammonia) are normal breakdown products of the digestion of protein. In hepatic encephalopathy (hepatic coma), toxins are not removed because liver function is impaired. Ammonia is produced by amino acid metabolism and intestinal urease-positive bacteria. In physiological conditions, it is mostly present as ammonium (NH4+) in serum. The urea or ornithine cycle, which is fully expressed in the liver exclusively, serves to converts NH4+ to urea prior to renal excretion and to maintain low serum concentrations. In hepatic coma, when the liver cannot remove toxins and urea cycle is not functional, all this occurs:

NH3 + α-ketoglutarate → Glutamate

α-ketoglutarate is used up which leads to:

·        ↑glutamate → ↑GABA (inhibitory neurotransmitter)

·        Inhibition of citric acid cycle/tricarboxylic acid cycle; this causes impairment of ATP formation.

·        Inhibition of metabolism of amino acids (impairment of transamination reactions).

 

NH3 + Glutamate → Glutamine

Glutamine is an amide of glutamic acid which provides a non-toxic storage and transport form of ammonia (NH3). Ammonia increase synthesis of glutamine in brain. Accumulation of glutamine in brain results in elevation of osmotic pressure in nervous cells leading to brain edema.

                                   

                                    NH3 + H+ → NH4+

In blood ammonia (NH3) is represented as ammonium ion (NH4+). Accumulation of ammonium ion impairs transport of ions (Na+, K+) through cell membranes and failure of transmission of nerve impulse.

            Urea cycle takes place exclusively in the liver, so in hepatic coma, urea level is low. Glutamine toxicity in brain is dependent on increased ammonia concentration.

Bilirubin toxicity will most likely be related to increase hemolysis, which is not the case in this question. Histamine is a biogenic amine produced from the amino acid histidine.

128. A patient with urolithiasis has developed severe pain attacks. For pain shock prevention he was administered an antispasmodic narcotic analgesic along with atropine. Name this drug:

Explanation

All options listed are all opoid analgesics; but Promedol remains the best answer because it has a spasmolytic effect (antispasmodic). Therefore, it is very effective for pains associated with spasms of smooth muscles of internal organs and blood vessels. In comparison with morphine hydrochloride, it has less oppressing activity on respiratory centers, less than excites the center of the vagus nerve and the vomiting center. When the pain is associated with spasms of smooth muscle (angina pectoris, liver, kidney, intestinal cramps) you can assign promedol with atropine, metacin, papaverine etc.

129.

A patient with acute myocardial infarction has been administered heparin as a part of complex therapy. Some time after heparin injection the patient developed hematuria. What heparin antagonist should be injected to remove the complication?

Explanation

Heparin is a natural anticoagulant produced in mast cells and basophils. It is an injectable, rapidly acting anticoagulant that is often used acutely to interfere with the formation of thrombi. Heparin is used in the prevention of venous thrombosis and the treatment of a variety of thrombotic diseases such as pulmonary embolism and acute myocardial infarction. Heparin binds to antithrombin III, with the subsequent rapid inactivation of coagulation factors. Antithrombin III inhibits serine proteases, including several of the clotting factors, most importantly, thrombin (factor IIa) and Factor Xa (a-active).

Protamine sulphate antagonizes the anticoagulant effects of heparin. The positively charged protamine interacts with the negatively charged heparin forming a stable complex without anticoagulant activity.

130.

A 37-year-old woman complains of headache, vertigo, troubled sleep, numbness of limbs. For the last 6 years she has been working at a gas-discharge lamp-producing factory in a lead-processing shop. Blood test findings: low hemoglobin and RBC level, serum iron concentration exceeds the norm by several times. Specify the type of anemia:

Explanation

FullSizeRender (6)

Lead poisoning: lead inhibits Ferrochelatase and ALA dehydratase. Ferrochelatase catalyzes Protoporphyrin + Fe+ → Heme

ALA dehydratase catalyzes Aminolevulinic acid (ALA) → Porphobilinogen

Therefore, Iron, Protoporphyrin and ALA will be accumulated in the body. Iron refractory anemia results from disorder of Iron inclusion in heme at decrease of enzymes activity, which catalyzes synthesis of heme.

131.

Despite the administration of cardiotonics and thiazide diuretic a patient with chronic heart failure has persistent edemas and the risk of ascites arose. What medication should be administered to enhance the diuretic effect of the administered drugs?

Explanation

IMG_9896

K+ sparing diuretics:

  - aldosterone antagonist: spironolactone, eplerenone

  - inhibit Na+ reabsorption: triamterene

The answer is Spironolactone because after the administration of cardiotonics and a thiazide diuretic, if the edema still persists, it is probably due to an increased action of aldosterone on the principal cells in collecting tubule of the kidney. In this case, an aldosterone antagonist will enhance the diuretic effect to prevent the occurrence of ascites. Blocking aldosterone effect prevent further reabsorption of Na+ and H2O which will definitely enhance the diuretic effect. Aldosterone acts on mineralocorticoid receptor → mRNA → protein synthesis (synthesis of Na+ channels). Spironolactone prevents/blocks the synthesis of Na+ channels.

132. Acute renal impairment caused death of a patient with hemorrhage. Autopsy revealed enlarged kidneys with broad pale-pink cortical layer expressively demarcated from dark-red renal pyramids. Macroscopic examination revealed lack of epithelial nuclei of convoluted tubules, tubulorrhexis, phlebostasis. The cell nuclei of choroid glomus and straight tubules were present. What pathology is it?

Explanation

     Acute tubular necrosis/Necrotic nephrosis/Necronephrosis involves the death of tubular epithelial cells that form the renal tubules of the kidneys. Most common cause of acute renal failure. It can be ischemic or nephrotoxic.

·        Ischemic acute tubular necrosis occurs due to hypoperfusion of the kidneys.

·        Nephrotoxic acute tubular necrosis occurs as a result of direct damage to tubular cells by ingestion, injection or inhalation of a number of toxic agents. Toxic agents causing nephrotoxic acute tubular necrosis includes mercuric chloride, ethylene glycol, carbon tetrachloride etc.

Macroscopically, the kidneys are enlarged and swollen (edematous). On cut section, the cortex is pale, while the medulla is slightly darker than normal. The capsule can be easily removed.

133.

A 3-year-old child with meningeal symptoms died. Postmortem macroscopy of the pia matter revealed miliary nodules which were microscopically represented by a focus of caseous necrosis with masses of epithelioid and lymphoid cells with large cells containing crescent-shaped peripheral nuclei situated between them. Specify the type of meningitis in the child:

Explanation

FullSizeRender (7)

When tubercle bacilli are introduced into the tissue, they are ingested by the alveolar macrophage. Macrophages undergo changes resembling epithelial cells – EPITHELOID cells. Some of the macrophages form MULTINUCLEATED GIANT cells by fusion of adjacent cells (langerhan’s or foreign body type). The giant cells may have 20 or more nuclei. These nuclei may be arranged at the periphery like HORSE-SHOE, RING or clustered at the poles or they may be present centrally (foreign body giant cells). Lymphocytes, plasma cells and fibroblasts surround the epitheloid cells and giant cells (hard tubercle- no central necrosis). Within 10-14 days, the centre of the cellular mass begins to undergo caseation necrosis – soft tubercle. This is the hallmark of tuberculous lesions.

FullSizeRender (9)

During the hematogenous spreading, the most serious immediate complication is MILIARY tuberculosis. The name miliary derives from its resemblance to millet seeds. Lesions of miliary tuberculosis consist of small granulomas, with a central necrotic portion. Organs often affected are lung, spleen, liver, kidney, meninges and bone marrow.

134. Monoamine oxidase inhibitors are widely used as psychopharmacological drugs. They change the level of nearly all neurotransmitters in synapses, with the following neurotransmitter being the exception:

Explanation

image

Monoamine oxidase (MAO) is a mitochondrial enzyme found in nerve and other tissues, such as the gut and liver. In the neuron, MAO functions as a “safety valve” to oxidatively deaminate and inactivate any excess neurotransmitter molecules (norepinephrine, dopamine and serotonin) that may leak out of synaptic vesicles when the neuron is at rest.

The MAO inhibitors (MAOIs) may irreversibly or reversibly inactivate the enzyme, permitting neurotransmitter molecules to escape degradation and therefore, to both accumulate within the presynaptic neuron and leak into the synaptic space. This is believed to cause activation of norepinephrine and serotonin receptors, and it may be responsible for the indirect antidepressant action of these drugs. Four MAOIs are currently available for treatment of depression: phenelzine, tranylcypromine, isocarboxazid, selegiline.

135. During histological analysis of the lymph node situated in the posterior neck triangle of an 18-year-old patient a morphologist detected a cluster of cells including the following: isolated multinucleate Reed-Sternberg cells, large and small Hodgkin’s cells and numerous lymphocytes, isolated plasma cells, eosinophils. What disease has developed in the patient?

Explanation

FullSizeRender (28)    

 Hodgkin’s disease or lymphogranulomatosis is a disorder involving primarily the lymphoid tissue. It is separated from non-hodgkin lymphoma because:

First, it is characterized morphologically by the presence of distinctive neoplastic giant cells called Reed-sternberg’s cells, admixed with a variable inflammatory infiltrate.

Secondly, it is often associated with distinctive clinical features, including systemic manifestations such as fever.

Reed-sternberg cell: classically, it is a large cell, most often binucleate or bilobed, with two halves often appearing as mirror images of each other. Its identification is essential for the histologic diagnosis. The origin of Hodgkin’s lymphoma is unknown.

136.

An infant has pylorospasm, weakness, hypodynamia, convulsions as a result of frequent vomiting. What kind of acid-base disbalance is it?

Explanation

Acidosis is the reduction in pH (increase in H+ concentration) below normal range. pH is less than 7.35; it is produced by:

·        Increase in partial pressure of CO2 in the body.

·        Decrease in HCO3- concentration.

Alkalosis is the increase in pH (decrease in H+ concentration) above normal range. pH is greater than 7.45; it is produced by:

·        Decrease in partial pressure of CO2 in the body.

·        Increase in HCO3- concentration.

Each of these two disorders has respiratory and non-respiratory forms. The non-respiratory form is divided into metabolic and excretory(non-gaseous).

·        Respiratory acidosis is the acidosis that is caused by alveolar hypoventilation e.g. airway obstruction due to bronchitis or lung diseases (pneumonia).

·        Respiratory alkalosis is caused by alveolar hyperventilation e.g. hypoxia in high altitude.

·        Non-respiratory:

-Metabolic acidosis is characterized by excess accumulation of organic acids such as lactic acid, ketoacids and uric acid formed by normal metabolism e.g. in Diabetes mellitus or extreme/prolonged exercise.

-Excretory/Non-gaseous acidosis may develop in impaired renal H+ excretion related to increased loss of bicarbonate in urine; diarrhea causes acidosis by the loss of bicarbonate with faeces.

-Excretory/Non-gaseous alkalosis: vomiting (loss of gastric acid), increased metabolism of lactate and citrate (turns into bicarbonate and water), long-term use of thiazides and loop diuretics.

      It is excretory/non-gaseous alkalosis because of the frequent vomiting.

137.

A 39-year-old man who had been operated for the stomach ulcer died 7 days after the surgery. Autopsy revealed that peritoneal leaves were dull, plephoric, covered with massive yellow-greenish films, the peritoneal cavity contained about 300 ml of thick yellow-greenish liquid. What pathologic process was revealed in the peritoneal cavity?

Explanation

Fibrinopurulent peritonitis: Supurative or purulent inflammation is characterized by the production of large amounts of pus. The cellular inflammatory response in peritoneal cavity is composed primarily of dense collections of neutrophils and fibrinopurulent debris that coat the visceral and abdominal wall. Serous peritonitis – thin fluid (not pus); Hemorrhagic peritonitis  - hemorrhage.

138.

Angiocardiography of a 60-year-old man revealed constriction of the vessel located in the left coronary sulcus of his heart. Name this pathological vessel:

Explanation

krushkrok No55 (2014)

* Ramus circumflexus (circumflex coronary artery) branch of left anterior descending artery runs within – left coronary sulcus.

* Arteria coronaria dextra (right coronary artery) – right coronary sulcus.

* Ramus interventricularis posterior (posterior interventricular branch): runs along the sulcus of the same name; greatest branch of right coronary artery

* Ramus interventricularis anterior (anterior interventricular branch): runs along the sulcus of the same name.

* Vena cordis parva (small cardiac veins): right portion of coronary sinus.

139.

A patient complains of pain in the right lateral abdomen. Palpation revealed a dense, immobile, tumor-like formation. The tumor is likely to be found in the following part of the digestive tube:

Explanation

imageimage

Anterolateral abdominal wall has 9 regions and 4 quadrants (RUQ, LUQ, RLQ, LLQ)

·        Right lateral abdominal region: Ascending colon (colon ascendens), right kidney, right ureter and loops of small intestine.

·        Umbilical region: Transverse colon (colon transversum), head of pancreas, duodenum (except superior part)

·        Left lateral abdominal region: Descending colon (colon descendens), left kidney, left ureter and loops of small intestine.

·        Left inguinal region: Sigmoid colon (colon sidmoideum), left ureter, left external iliac artery of artery and vein.

·        Right Inguinal region: Caecum, vermiform appendix, right ureter

140. During regular check-up a child is detected with interrupted mineralization of the bones. What vitamin deficiency can be the cause?

Explanation

image

Vitamin D is a fat soluble vitamin. Cholecalciferol (Vit. D3) is found in animals. It is regarded as a sunshine vitamin. Calcitriol is the biologically active form of Vit. D. Calcitriol acts at 3 different levels (intestine, kidney and bone) to maintain plasma calcium.

It increases the intestinal absorption of calcium and phosphate.

In the osteoblasts of bone, it stimulates calcium uptake for deposition as calcium phosphate. Thus, calcitriol is essential for bone formation.

Calcitriol is also involved in minimizing the excretion of calcium and phosphate through the kidney, by decreasing their excretion and enhancing reabsorption.

image

Deficiency of Vit D causes rickets in children and osteomalacia in adults. Rickets in children is characterized by bone deformities due to incomplete bone mineralization, resulting in soft and pliable bones and delay in teeth formation. The weight bearing bones are bent to form bow legs.

141. A patient suffers from intermittent fevers and normalizations of body temperature that occur during the day. The temperature rise is observed regularly every fourth day. Specify the type of temperature curve:

Explanation

krushkrok No50 (2014)

* Febris intermittens: characterized by regular alternation of brief attacks of fever (paroxysms) with feverless periods (apyrexia). Attacks occur every 3rd day, 2nd day or everyday. Increase temperature persists for several hours, drops to normal and then rises again.

* Febris continua/persistent: elevated temperature persists at a high level, difference between morning and evening temperature does not exceed 10C.

* Febris recurrens: characterized by longer periods of pyrexia than intermittent (5-6days). Question says every 4 days.

* Febris hectic: 3-50C (difference in temperature).

* Febris remittens: difference in temperature exceeds 10C but temperature never falls to normal.

142.

A woman with the III (В), Rh- blood group gave birth to a child with the II (А) blood group. The child is diagnosed with hemolytic disease of newborn caused by rhesus incompatibility. What blood group and Rh can the father have?

Explanation

image

Blood group O(I): no antigens, therefore no agglutination.

Blood group A(II): A antigen, agglutinate with blood group B(III) and O(I).

Blood group B(III): B antigen, agglutinate with blood group A(II) and O(I).

Blood group AB(IV): A and B antigen, agglutinate with all blood groups. No antibody.

Blood group name is determined by the antigen present on RBC, but the patient has an opposite antibody. So whenever, the antibody corresponds to the antigen, there is agglutination. Since O does not have any antigen, no agglutination can occur.

143. A patient is diagnosed with hereditary coagulopathy that is characterised by factor VIII deficiency. Specify the phase of blood clotting during which coagulation will be disrupted in the given case:

Explanation

144. A child is 10 years old. The following presentations have developed: sharp pain during swallowing, swollen neck, body temperature rise up to 39, 0oC , bright-red finely papular rash all over the body. Pharynx and tonsils are sharply hyperemic (\\\"flaming pharynx\\\"), \\\"crimson tongue\\\". On the tonsils surface there are isolated greyish necrosis focuses. What disease it might be?

Explanation

Scarlet fever is an acute infectious disease accompanied by local inflammatory changes mainly in the pharynx and typical generalized rash.

* Local changes: primary scarlatnic affect/complex – catarrhal or necrotic tonsillitis. Catarrhal tonsillitis: hyperemia of pharynx (flaring pharynx or burning faucet). Necrotic tonsillitis: coagulative necrosis and ulceration.

* General changes are first of all RASH. The face is also involved, but usually a small area about the mouth (nasolabial triangle) remains relatively unaffected to produce a circumoral pallor. Tongue is beefy red/crimson red and glistening, strawberry tongue.

145.

Along with normal hemoglobin types there can be pathological ones in the organism of an adult. Name one of them:

Explanation

image

Sickle cell hemoglobin (HbS) point mutation causes a single amino acid replacement in β chain of hemoglobin (substitution of glutamic acid with valine). Pathogenesis of sickle cell anemia: low O2, high altitude or acidosis precipitates sickling – modified shape (deoxygenated HbS polymerizes) → anemia and vaso-occlusive disease. Clinical findings include: dizziness, general weakness, fatigue, “crew-cut” on skull X-ray due to marrow expansion from increased erythropoiesis (this is also seen in thalassemia)  etc. Sickle cells are crescent-shaped RBCs.

146. Emotional stress causes activation of hormone-sensitive triglyceride lipase in the adipocytes. What secondary mediator takes part in this process?

Explanation

The mobilization of stored fat requires the hydrolytic release of fatty acids and glycerol from their triacylglycerol (TAG) form. This process is initiated by hormone-sensitive lipase, which removes a fatty acid from carbon 1 and/or carbon 3 of the TAG. This enzyme is activated when phosphorylated by cyclic AMP (cAMP)-dependent protein kinase. cAMP is produced in the adipocyte when one of several hormones (such as epinephrine or glucagon – stress hormones) binds to receptors on the cell membrane and activates adenylyl cyclase.

147.

A patient has been diagnosed with alkaptonuria. Choose an enzyme that can cause this pathology when deficient:

Explanation

image

Alkaptonuria: congenital deficiency of homogentisate oxidase (homogentisic acid oxidase) in the degradative pathway of tyrosine to Fumarate → pigment-forming homogentisic acid accumulates (homogentisuria) in tissues. Autosomal recessive. Usually benign. Urine turns black on prolonged exposure to air. May have debilitating arthralgias (homogentisic acid toxic to cartilage).

148. As a result of a continuous chronic encephalopathy a patient has developed spontaneous motions and disorder of torso muscle tone. These are the symptoms of the disorder of the following conduction tract:

Explanation

Rubrospinal tract: red nucleus – lateral white column (spinal cord); controls muscle tone, because of its connections with cerebellum, vestibular apparatus and skeletal muscle.

Corticospinal tract: anterior and lateral – control of voluntary movements. Forms upper motor neuron (UMN).

 Corticonuclear tract: innervates muscles of the face, head and neck. Spinothalamic tract: anterior – crude touch sensation; lateral – pain and temperature sensations.

Tectospinal tract: control of movement of head in response to visual and auditory impulse.

149.

For people adapted to high external temperatures profuse sweating is not accompanied by loss of large volumes of sodium chloride. This is caused by the effect the following hormone has on the perspiratory glands:

Explanation

Aldosterone produced in adrenal cortex (zona glomerulosa): causes increased sodium (Na+) reabsorption; increased potassium and hydrogen ions (↑K+, H+) excretion. They increase sodium (↑Na+) channel and Na+ /K+-pump insertion in principal cells of collecting duct; enhances K+ and H+ excretion by way of principal cell K+ channels and α-intercalated cell H+ ATPases of collecting duct. Therefore, increase in aldosterone → ↑ K+ in urine (excretion) and ↓ Na+ in urine (↑ reabsorption); And decreased aldosterone → ↓ K+ excretion (↓K+ in urine) and ↓ Na+  reabsorption (i.e. ↑Na+ in urine); same effects on sweats glands too.

150.

On examination of a newborn boy’s external genitalia a fissure in the urethra opening on the inferior surface of his penis is detected. What maldevelopment is it?

Explanation

image

Defects in male genitalia:

Hypospadia: fusion of the urethral folds is incomplete and abnormal openings of the urethra occur along the inferior (undersite) surface of the penis, usually near the glans, along the shaft or near the base of the penis.

Epispadia is a rare abnormality (1/30000 births) in which the urethral meatus is found on the dorsum (superior) surface of the penis.

151. Poisoning caused by mercury (II) chloride (corrosive sublimate) occurred in the result of safety rules violation. In 2 days the patient’s diurnal diuresis became 620 ml. The patient developed headache, vomiting, convulsions, dyspnea; moist crackles are observed in the lungs. Name this pathology:

Explanation

Acute tubular necrosis/Necrotic nephrosis/Necronephrosis involves the death of tubular epithelial cells that form the renal tubules of the kidneys. Most common cause of acute renal failure. It can be ischemic or nephrotoxic.

·        Ischemic acute tubular necrosis occurs due to hypoperfusion of the kidneys.

·        Nephrotoxic acute tubular necrosis occurs as a result of direct damage to tubular cells by ingestion, injection or inhalation of a number of toxic agents. Toxic agents causing nephrotoxic acute tubular necrosis includes mercuric chloride, ethylene glycol, carbon tetrachloride etc.

Macroscopically, the kidneys are enlarged and swollen (edematous). On cut section, the cortex is pale, while the medulla is slightly darker than normal. The capsule can be easily removed.

152.

It is known that the gene responsible for development of blood groups according to AB0 system has three allele variants. Existence of the IV blood group can be explained by the following variability form:

Explanation

     Type IV blood group – AB

Combinative because both alleles of this gene are equally expressed in dominant forms and there is no masking of the characteristics (phenotype) of any of the allele.

153.

When measuring power inputs of a person by the method of indirect calorimetry the following results were obtained: oxygen consumption is 1000 ml and carbon dioxide production is 800 ml per minute. The person under examination has the following respiratory coefficient:

Explanation

RQ = vCO2 ÷ vO2 = 800 ÷ 1000 (ml) = 0.8

Minute O2 uptake – 1000ml

    Minute CO2 emission – 800ml

    Respiratory Quotient (RQ) = vCO2 /vO2 = 800/1000(ml) = 0.8

154.

A 43-year-old-patient has arterial hypertension caused by increase in cardiac output and general peripheral resistance. Specify the variant of hemodynamic development of arterial hypertension in the given case:

Explanation

Eukinetic: Increase in both systolic and diastolic blood pressure.

       Hyperkinetic: Increase in systolic blood pressure.

       Hypokinetic: Increase in diastolic blood pressure.

       Cardiac output determines systolic blood pressure; peripheral resistance determines diastolic blood pressure.

155. A patient has been hospitalised with provisional diagnosis of virus B hepatitis. Serological reaction based on complementation of antigen with antibody chemically bound to peroxidase or alkaline phosphatase has been used for disease diagnostics. Name this serological reaction:

Explanation

Immune-enzyme analysis (enzyme-linked immunosorbent assays): used to detect the presence of either a specific antigen or a specific antibody in a patient’s blood sample. Common enzymes used are peroxidase, alkaline phosphatase or glucose oxidase. Producing an observable colour change or fluorescence. When radioactive isotopes are incorporated – Radioimmunoassay.

156.

A surgeon has to find the common hepatic duct during operative intervention for treatment of concrements in the gall ducts. The common hepatic duct is located between the leaves of:

Explanation

krushkrok No95a (2012)     

Hepatoduodenal ligament running from the porta hepatica to the superior part of the duodenum. It consists of double layer of peritoneum and enfolds 3 structures:

·        Common hepatic artery

·        Portal vein

·        Common bile duct

Portal vein is localized posteriorly; hepatic artery – left anterior and bile duct – right anterior.

157.

Cells of healthy liver actively synthesize glycogen and proteins. What organelles are the most developed in them?

Explanation

The network of membrane enclosed spaces that extends throughout the cytoplasm constitutes endoplasmic reticulum (ER). Some of these thread-like structures extend from the nuclear pores to the plasma membrane.

A large portion of the ER is studded with ribosomes to give a granular appearance which is referred to as rough endoplasmic reticulum (rER). Ribosomes are the factories of protein biosynthesis. The smooth ER (agranular ER) does not contain ribosomes. It is involved in the synthesis of lipids (triacylglycerols, phospholipids, sterols) and metabolism of drugs, besides supplying Ca2+ for the cellular functions.

158.

Atria of an experimental animal were superdistended with blood, which resulted in decreased reabsorption of N a+ and water in renal tubules. This can be explained by the influence of the following factor on kidneys:

Explanation

Atrial natriuretic peptide is secreted in response to increase atrial pressure. It causes increase glomerular filtration rate (GFR) and increase sodium ion filtration with no compensatory sodium ion reabsorption and water in distal nephron which leads to increase diuresis.

   Antidiuretic hormone (vasopressin) is secreted in response to decrease blood volume and increase plasma osmolarity. It binds to receptors on principal cells of collecting ductules causing increase number of aquaporins and increase water reabsorption which leads to decreased diuresis.

 

159. A patient with hypertensic crisis has increased content of angiotensin II in blood. Angiotensin pressor effect is based on:

Explanation

FullSizeRender (12)

↓blood supply → activates renin. Renin catalyzes the conversion of angiotensinogen to angiotensin I. Angiotensin converting enzyme (ACE) converts angiotensin I → angiotensin II. Angiotensin II causes:

- vasoconstriction of arteriolar muscles → ↑BP

- vasopressin (ADH) → ↑H2O reabsorption → ↑plasma volume

- aldosterone → ↑Na+ and H2O reabsorption → ↑plasma volume → ↑BP

160.

A 4-year-old child was admitted to an orthopaedic department with displaced shin fracture. Bone fragments reposition requires analgesia. What drug should be chosen?

Explanation

All options listed are all opoid analgesics (except panadol); but Promedol remains the best answer because it has a spasmolytic effect (antispasmodic). Therefore, it is very effective for pains associated with spasms of smooth muscles of internal organs and blood vessels. In comparison with morphine hydrochloride, it has less oppressing activity on respiratory centers, less than excites the center of the vagus nerve and the vomiting center. When the pain is associated with spasms of smooth muscle (angina pectoris, liver, kidney, intestinal cramps) you can assign promedol with atropine, metacin, papaverine etc.

161. Doctors make mostly radial incisions during mammary gland surgery. What particulars of anatomical organization make such surgical technique preferable?

Explanation

Mammary gland: The nipple (papilla mammae) is situated on the anterior surface of the gland. It is surrounded by the areola (areola mammae). The areola contains the areolar (Montgomery’s) glands and sebaceous glands. The mammary gland is composed of acini, which make up the lobules, whose aggregation will form the lobes of the gland. The lobes are arranged in a radiating fashion and drained into lactiferous ducts. 18-20 lobes drain through 18-20 ducts, which converge or open on the nipple.

The lobes are partly separated from each other by irregular and incomplete fibrous septa, which are continuous with the gland stroma. The organ is anchored to the overlying skin and the underlying pectoral fascia by bands of fibrous tissue called the Cooper’s ligaments.

162. A 41-year-old man has a history of recurrent attacks of heartbeats (paroxysms), profuse sweating, headaches. Examination revealed hypertension, hyperglycemia, increased basal metabolic rate, and tachycardia. These clinical presentations are typical for the following adrenal pathology:

Explanation

FullSizeRender (22)

Adrenal medulla produces catecholamines (epinephrine, norepinephrine and dopamine). Increased secretion of epinephrine & norepinephrine can result in these symptoms listed. Since the patient is 41yrs old, this can be pheochromocytoma - if it is a tumor.

163.

A 12-year-old child has developed nephritic syndrome (proteinuria, hematuria, cylindruria) 2 weeks after the case of tonsillitis, which is a sign of affected glomerular basement membrane in the kidneys. What mechanism is the most likely to cause the basement membrane damage?

Explanation

FullSizeRender (17)

Tonsillitis is commonly caused by both aerobic and anaerobic bacteria (streptococcus, staphylococcus and haemophilus).

Acute poststreptococcal glomerulonephritis: most frequently seen in children. Occurs approximately 2 weeks after group A streptococcal infection of pharynx or skin. Resolves spontaneously. Type III hypersensitivity reaction (Immune complex). Presents with peripheral and periorbital edema, cola-coloured urine (hematuria), hypertension, proteinuria. On immunofluorescent microscopy: granular appearance due to IgG, IgM and C3 deposition along glomerular basement membrane and mesangium. On electron microscopy: subepithelial immune complex humps. On light microscopy: glomeruli enlarged and hypercellular.
164. Several minutes after a dentist administered novocaine for local anaesthesia of a patient’s tooth, the following symptoms sharply developed in the patient: fatigue, skin itching. Objectively the following can be observed: skin hyperemia, tachycardia, BP dropped down to 70/40 mm Hg. What kind of allergic reaction is this pathology?

Explanation

FullSizeRender (17)

Type I Hypersensitivity reaction (HSR); Immediate; Reagin; anaphylactic and atopic: free antigen cross-links IgE on presensitized (i.e. exposed to the antigen before) mast cells and basophils, triggering immediate release of vasoactive amines that act at postcapillary venules (i.e. histamine). Reaction develops rapidly after antigen exposure (after novocaine exposure) because of preformed antibody from first exposure. IgE is the main immunoglobulin involved in type I HSR. Type I: uses IgE and IgG4

165.

A patient with probable liver abscess was delivered to a surgical department. The patient for a long time had been on an assignment in an African country and had recurrent cases of acute gastrointestinal disturbance. What protozoan disease can it be?

Explanation

image

Entamoeba Histolytica causes amebic dysentery and liver abscess. The lifecycle of E. histolytica has two stages: the motile ameba (trophozoite) and the non-motile cyst. The cyst predominates in non-diarrheal stools. The cyst has 4 nuclei, an important diagnostic criterion. The organism is acquired by ingestion of cysts that are transmitted primarily by the fecal-oral route in contaminated food and water.

krushkrok No14 (2012)

Amebic abscess of the liver is characterized by right-upper-quadrant pain, weight loss, fever and a tender enlarged liver. Right-lobe abscess can penetrate the diaphragm and cause lung disease. Most cases of amebic liver abscess occur in patients who have not had overt intestinal amebiasis.

166. Work in a mine is known to cause inhalation of large amounts of coal dust. Inhaled coal dust can be detected in the following pulmonary cells:

Explanation

Alveolar macrophages remove inhaled particulate matter from the air spaces and red blood cells (RBCs) from the septum of alveoli. They are unusual in that they function both in the connective tissue of the septum (alveolar wall) and in the air space of the alveolus (surface of alveolar cells). In air spaces they scavenge the surface to remove inhaled particulate matter (e.g. dust, pollen, pathogens), thus giving them one of their alternate names – Dust cells. They also phagocytose infectious organisms such as Mycobacterium tuberculosis. Other macrophages remain in the septal connective tissue, where, filled with accumulated phagocytized material, they may remain for much of an individual’s life. Thus, at autopsy, the lungs of urban dwellers as well as smokers will usually show many alveolar and septal macrophages filled with carbon particles, anthracotic pigment and birefringent needle-like particles of silica.

Type I alveolar cells (type I pneumocytes): the junctions formed between this cells form an effective barrier between the air space and the components of the septal wall. They are not capable of cell division. They make up 95% of the surface of the alveoli.

Type II alveolar cells: secretory cells; have lamellar bodies; progenitor cells for type I alveolar cells.

Clara cells: non-ciliated; low columnar/cuboidal cell with secretory granules. They secrete components of surfactant, degrade toxins and act as reserve cells.

Endothelial cells line blood vessels.

167.

Electrical activity of neurons is being measured. They fire prior to and at the beginning of inhalation. Where are these neurons situated?

Explanation

Respiratory centers are group of neurons which control the rate, rhythm and force of respiration. Depending upon the situation in brainstem, the respiratory centers are classified into two groups:

·         Medullary centers consisting of:       * dorsal respiratory group of neurons

                                                                       *ventral respiratory group of neurons

·         Pontine centers:           * apneustic center

                                                 *pneumotaxic center

Dorsal respiratory group of neurons are diffusely situated in the nucleus of tractus solitaries which is present in the upper part of the medulla oblongata. Usually these neurons are collectively called inspiratory center. All neurons of the dorsal respiratory group are inspiratory neurons and generate inspiratory ramp by the virtue of their autorhythmic property. They are responsible for the basic rhythm of respiration.

Ventral respiratory group of neurons are present in nucleus ambiguous and nucleus retroambiguous in the medulla oblongata, anterior and lateral to the nucleus of tractus solitaries. Ventral respiratory group has both inspiratory and expiratory neurons.

168. Electrocardiogram analysis demonstrates that cardiac cycle of a human equals 1 second. It means that heart rate per minute equals:

Explanation

Cardiac cycle is defined as the succession of (sequence of) coordinated events taking place in the heart during each beat (during 1 heart beat). Therefore, if 1 cardiac cycle (i.e. 1 heart beat) equals 1 second; then heart rate per minute will be 60 (1 minute = 60secs).

Each heart beat consist of two major periods called systole and diastole. During systole, heart contracts and pumps the blood through arteries. During diastole, heart relaxes and blood is filled in the heart. All these changes are repeated during every heart beat in a cyclic manner.

169. Glomerular filtration of a person, who has been starving for a long time, has increased by 20%. The most likely cause of filtration changes in the given conditions is:

Explanation

     Starvation decreases protein synthesis because the substrates are not readily available. Decrease in protein synthesis decreases oncotic pressure and thereby increasing glomerular filtration. Net filtration pressure is the balance between hydrostatic pressure (glomerular capillary pressure) and oncotic pressure.

Net filtration pressure = Hydrostatic pressure – Oncotic pressure

Therefore, a decrease in oncotic pressure increases the filtration rate according to the degree of starvation which decreases protein synthesis.

170.

Analysis of sputum taken from a patient with suspected pneumonia revealed slightly elongated gram-positive diplococci with pointed opposite ends. What microorganisms were revealed in the sputum?

Explanation

     There are two medically important genera of gram positive cocci: staphylococcus and streptococcus. Microscopically, staphylococci appear in grape-like clusters, whereas streptococci are in chains or pairs. Streptococcus pneumonia causes pneumonia, bacteremia, meningitis and infections of the upper respiratory tract. Pneumococci are gram positive lancet-shaped cocci arranged in pairs (diplococci) or short chains. The term “lancet-shaped” means that the diplococci are oval with somewhat pointed ends rather than being round).

Klebsiella and the Neisseria genus are gram negative.

171. Serological diagnostics of infectious diseases is based upon specific interaction with antigenes. Specify the serological reaction that underlies adhesion of microorganisms when they are affected by specific antibodies in presence of an electrolyte:

Explanation

     Agglutination is commonly used as a method of identifying specific bacterial antigens and in turn the identity of such bacteria. Because the clumping reaction occurs quickly and is easy to produce, agglutination is an important technique in diagnosis. It is the clumping of cells such as bacteria or red blood cells in the presence of an antibody or complement. The antibody or other molecule binds multiple particles and joins them creating a large complex. The antibody (agglutinin) and antigen (agglutinogen) take part in the agglutination reaction. Their interaction takes place in definite quantitative proportions and in the presence of an electrolyte (0.85% NaCl solution). In mechanism and outer manifestation, the agglutination reaction is similar to the precipitin reaction. Both reactions are accompanied by the production of visible precipitates of antigen with the difference that in the agglutination reaction microbial bodies serve as the antigen, whereas in the precipitin reaction (precipitation reaction) the antigen is the product of the breakdown of microbial bodies, very minute particles of dissolved antigens requiring a large amount of antibodies for complete interaction.

172.

Autopsy of a man who had tuberculosis revealed a 3x2 cm large cavity in the superior lobe of the right lung. The cavity was interconnected with a bronchus, its wall was dense and consisted of three layers: the internal layer was pyogenic, the middle layer was made of tuberculous granulation tissue and the external one was made of connective tissue. What is the most likely diagnosis?

Explanation

Secondary tuberculosis usually results from reactivation of dormant, endogenous tubercle bacilli in a sensitized patient who has had previous contact with the tubercle bacillus. Reactivation typically begins in the apical or posterior segments (often 1st and 2nd segments) of one or both upper lobes (“simon’s foci”), where the organisms were seeded during the primary infection. There are 8 forms or stages of the secondary tuberculosis: Acute local tuberculosis; Fibrous local tuberculosis; Infiltrative tuberculosis; Tuberculoma; Caseous pneumonia; Acute cavernous tuberculosis; Fibrous cavernous tuberculosis; Cirrhotic tuberculosis.

Fibrous cavernous tuberculosis is the most frequent form. Macroscopically, the lesions are spherical and cavitary (cavity can contain blood and blood clots); the so-called coin lesions. Microscopically, the outer wall of cavity shows fibrosis or sclerosis. Internal surface may be connected with bronchus. The wall of cavern has 3 membranes:

·        Internal membrane occurs by necrotic tissue

·        Middle membrane occurs by special granular tissue

·        External membrane occurs by connective fibrous tissue

173.

A 7-year-old child has acute onset of disease: temperature rise up to 38oC , rhinitis, cough, lacrimation, and large-spot rash on the skin. Pharyngeal mucosa is edematous, hyperemic, with whitish spots in the buccal area. What kind of inflammation causes the changes in the buccal mucosa?

Explanation

Catarrhal inflammation is one of the morphologic patterns in acute inflammation. In this type, a surface inflammation is associated with greatly increased secretion of clear mucus (nasal discharges). Rhinitis is inflammation of mucous lining of the nose.

174.

A 65-year-old man suffering from gout complains of pain in his kidneys. Ultrasonic examination revealed kidney stones. A certain substance in increased concentration can cause kidney stones formation. Name this substance:

Explanation

image

The end product of the purine nucleotides catabolism in humans and other primates is uric acid (urate) which is excreted in urine. Allopurinol and febuxostat inhibits Xanthine oxidase (XO). Hypoxanthine and Xanthine which is more soluble is excreted in urine. Purine nucleotides (adenine and guanine).  AMP – Adenosine monophosphate; GMP – Guanosine monophosphate

Further notes: krushkrok No21 (2014) Gout is a metabolic disorder characterized by high levels of uric acid in the blood. Hyperuricemia can lead to deposition of sodium urate crystals (uric acid salts) in tissues, especially the joints and kidney (forming kidney stones). Hyperuricemia does not always lead to gout, but gout is always preceded by hyperuricemia. In humans, sodium urate is the end product of purine metabolism. The deposition of urate crystals initiates an inflammatory process involving the infiltration of granulocytes that phagocytize the urate crystals. This process generates oxygen metabolites, which damage tissues, resulting in the release of lysosomal enzymes that evoke an inflammatory response. The cause of hyperuricemia is an overproduction of uric acid relative to the patient\\\'s ability to excrete it. Most therapeutic  strategies for gout involve lowering the uric acid level below the saturation point (below 6mg/dL), thus preventing the deposition of urate crystals.
175.

Sex chromatin was detected during examination of a man’s buccal epithelium. It is characteristic of the following chromosome disease:

Explanation

FullSizeRender (1)

krushkrok No70 (2007)

A man (XY) with an additional sex chromatin (X-chromosome) – XXY (Klinefelter’s syndrome). Causes :

* nondisjunction (maternal and paternal nondisjunction in meiosis I)

* Mosaicism: with the karyotype being 46, XY/47, XXY

Manifestations: gynecomastia, female pattern of pubic hair distribution, no facial hair, high voice.

Turner\'s syndrome (XO) and Triple X syndrome (XXX)  relates to females ONLY! The other options are not connected to sex chromosomes.
176. Pure culture of microorganisms was obtained from pharynx of a child with suspected diphtheria. Morphologic, tinctorial, cultural, and biochemical properties of the microorganisms were studied and revealed to be characteristic of diphtheria agents. What investigation should be additionally performed to make a conclusion, that these microorganisms are pathogenic diphtheria bacilli?

Explanation

Having carried out all these examinations (morphological, tinctural, cultural and biochemical), then to draw a conclusion on its pathogenicity (i.e. its ability to cause disease), another examination determining the toxigenicity of this organism needs to be performed. Because the exotoxins produced by corynebacterium diphtheria cause the disease manifestations. Exotoxin production is essential for pathogenesis. Diphtheris toxin inhibits protein synthesis by ADP-ribosylation of elongation factor 2 (EF-2). The toxin affects all eukaryotic cells.

177.

A group of Ukrainian tourists returning from Samarqand was bringing with them gerbils. During examination in customs office ulcers were detected on the skin of the animals. What protozoa is the most likely to cause the disease in the animals, if mosquitoes are the carriers?

Explanation

The genus Leishmania includes four major pathogens: Leishmania donovani, Leishmania tropica, Leishmania mexicana and Leishmania braziliensis. L. tropica and mexicana cause cutaneous leishmaniasis; the former organism is found in the old world, whereas the latter is found only in the Americas. The lesions are confined to the skin in cutaneous leishmaniasis. A granulomatous response occurs and a necrotic ulcer forms at the bite site. The lesions tend to become superinfected with bacteria. Old world cutaneous leishmaniasis (oriental sore, Delhi boil), caused by L. tropica, is endemic in the Middle East, Africa and India. image The initial lesion of cutaneous leishmaniasis is a red papule at the bite site, usually on an exposed extremity. This enlarges slowly to form multiple satellite nodules that coalesce and ulcerate.
178.

A 5-year-old child has been diagnosed with acute right distal pneumonia. Sputum inoculation revealed that the causative agent is resistant to penicillin and sensitive to macrolides. What drug should be prescribed?

Explanation

     Macrolides inhibit protein synthesis by blocking translocation, they bind to the 23S rRNA of the 50S ribosomal subunit. It is bacteriostatic. Examples of macrolides includes azithromycin, clarithromycin, erythromycin etc.

All other options are not macrolides, but are antibiotics.

Ampicillin is an extended spectrum penicillin; a β-lactam antibiotic.

Gentamycin and streptomycin are Aminoglycosides. Together with tetracyclines, they are 30S ribosomal subunit inhibitor.

179.

To an emergency ward a 7-year-old child was delivered in the condition of allergic shock caused by a bee sting. High concentration of histamine is observed in blood. Production of this amine is the result of the following reaction:

Explanation

image     

Certain amino acids undergo decarboxylation that means the removal of their α-carboxyl group resulting in liberation of CO2 and formation of biogenic amines. Biogenic amines are physiologically active substances such as hormones, neurotransmitters etc. decarboxylation of amino acids:

·        Tryptophan → Niacin → NAD+/NADP+

         Tryptophan → Serotonin →Melatonin

·        Histidine → Histamine

·        Glutamine → GABA

         Glutamine → Glutathione

180.

After implantation of a cardiac valve a young man systematically takes indirect anticoagulants. His state was complicated by hemorrhage. What substance content has decreased in blood?

Explanation

     Indirect anticoagulants:

·        Coumarine derivatives – Neodicumarin, warfarin, syncumar

·        Indandione derivatives – Phenylinum

They antagonize the cofactor functions of vitamin K. Vitamin K is essential for the formation of various clotting factors in the liver, namely clotting factor II, VII, IX and X; Protein C and S.

Factor II – Prothrombin

Factor VII – Stable factor

Factor IX – Christmas factor

Factor X – Stuart-Prower factor

These factors undergo vitamin K-dependent post-translational modification, whereby a number of their glutamic acid residues are carboxylated to form ɣ-carboxyglutamic acid residues. The ɣ-carboxyglutamyl residues bind calcium ions which are essential for interaction between the coagulation factors and platelet membranes. This oral anticoagulants block epoxide reductase and creation of active form of vitamin K resulting in disturbances in prothrombin and proconvertin synthesis in liver.

All other options are not clotting factors.

181. A 12-year-old patient has white non-pigmented spots on the skin. The spots appeared after the patient became 10 years old, and they constantly grow. This spots appeared due to the lack of the following skin cells:

Explanation

Melanin is a class of organic compounds found in plants, animals, protozoa and bacteria, where they mainly play the role of pigments. Chemically, these compounds are derivatives of amino acid tyrosine. The most common in humans are eumelanin and pheomelanin. The synthesis of melanin in the skin i.e. melanogenesis, is stimulated by ultraviolet (UV) radiation impact on the DNA. Melanogenesis occurs in melanocytes i.e. specialized dendritic cells. During embryonic development melanocytes precursors arise from the neural crest and actively migrate to the peripheral zones. Epidermis basal layer cells contain melanin pigment grains. These grains are also present in melanin pigment cells of papillary dermis and they are called melanoblasts. The more pigment inclusions, the darker the colour of the skin. In people with dark skin, Asians and Afro-Americans, pigment inclusions are found in the cells of spinous and granular layers of epidermis. Depigmentation (Non-pigmentation) producing white spots (macula) due to lack of melanin pigments in melanocytes can be: general - albinism; local - vitiligo; leukoderma
182.

A 16-year-old adolescent is diagnosed with hereditary UDP (uridine diphosphate) glucuronyltransferase deficiency. Laboratory tests revealed hyperbilirubinemia caused mostly by increased blood content of the following substance:

Explanation

image

In the hepatocyte, the solubility of unconjugated bilirubin is increased (i.e. it is made soluble) by the addition of two molecules of glucuronic acid to produce conjugated bilirubin. This process is referred to as conjugation. This reaction is catalyzed by Uridine diphosphate (UDP) glucuronyltransferase – UGT. Varying degrees of deficiency of this enzyme result in Crigler-Najjar I & II and Gilbert syndrome; with Crigler-Najjar I being the most severe deficiency.

Deficiency of UGT inhibits conjugation and therefore increase unconjugated bilirubin in serum (Jaundice).

183.

Prior to glucose utilization in cells it is transported inside cells from extracellular space through plasmatic membrane. This process is stimulated by the following hormone:

Explanation

In the liver and muscle, insulin increases glycogen synthesis. In the muscle and adipose tissue, insulin increases glucose uptake by increasing the number of glucose transporters (GLUT-2) in the cell membrane. Insulin is an anabolic hormone; it increases glucose uptake, glycogen synthesis, protein synthesis, fat synthesis; and decrease gluconeogenesis, glycogenolysis, lipolysis. 

Insulin facilitates the transport of glucose and amino acids into target organs: In liver – insulin increases the storage of glucose as glycogen. It decreases protein catabolism; In muscles – it stimulates glycogen synthesis and protein synthesis; In Adipose tissue – it facilitates triglyceride storage by activating plasma lipoprotein lipase which increase glucose transport into the cell and by reducing intracellular lipolysis.

Glucagon is a functional antagonist of insulin.

184. Atretic bodies and developed yellow body can be observed along with follicles of various orders in an ovary specimen. What stage of ovarian and menstrual cycle is characterized by the described ovary condition?

Explanation

In the course of a normal menstrual cycle, the ovary will go through 3 phases: follicular phase, ovulation and luteal phase. Also, during the menstrual cycle, the uterine endometrium passes through 3 stages: the proliferative phase, secretory or progestational phase and the Menstrual phase. At the beginning of each ovarian cycle, 15 to 20 primary-stage (preantral) follicles are stimulated to grow under the influence of follicle stimulating hormone (FSH). Under normal conditions, only one of these follicles reaches full maturity and only one oocyte is discharged; the others degenerate and become atretic. When a follicle becomes atretic, the oocyte and surrounding follicular cells degenerate and are replaced by connective tissue, forming a corpus atreticum (atretic body). During the luteal phase, after the release of the oocyte, the remaining granulosa and theca cells on the ovary form the corpus luteum. The granulosa cells have a vacuolated appearance with accumulated yellow pigment, hence the name corpus luteum ('yellow body'). If fertilization does not occur when the oocyte is released, shedding of the endometrium (compact and spongy layers) marks the beginning of the menstrual phase. When the menstrual phase begins, blood escapes from superficial arteries and small pieces of stroma and glands break away. During the following 3 or 4 days the compact and spongy layers are expelled from the uterus and the basal  layer is the only part of the endometrium that is retained. Therefore, it is premenstrual because the atretic bodies and yellow body can be observed before the menstrual phase.
185. A 46-year-old female patient has continuous history of progressive muscular (Duchenne’s) dystrophy. Which blood enzyme changes will be of diagnostic value in this case?

Explanation

Creatine kinase/Creatine phosphokinase is an enzyme present in striated muscles, used to generate energy. When its serum level is elevated it is often an indication of muscle damage caused by injury, muscular dystrophy or cardiac problems. Most forms of muscular dystrophy are associated with decreased creatinine excretion. Creatinine is a break down product of creatine. Normal constituent of urine but can be elevated in muscular dystrophy. Since creatine is not a normal constituent of urine, it is more specific in this case.

186.

A laboratory experiment on a dog was used to study central parts of auditory system. One of the mesencephalon structures was destroyed. The dog has lost the orienting response to auditory signals. What structure was destroyed?

Explanation

     Quadritubercular bodies/tectal plate is the dorsal portion of the midbrain comprises 4 colliculi – two superior and two inferior. The nuclei of colliculi are responsible for reflexes associated with sudden sound and visual stimuli (auditory and visual orientative reflexes); they also maintain consciousness. The nuclei give rise to the tectospinal tract. Superior colliculi – visual orientative reflex; Inferior colliculi – auditory orientative reflex.

187.

A patient has decreased concentration of magnesium ions that are required for ribosomes connection to granular endoplasmic reticulum. This condition is known to disrupt the process of protein biosynthesis. Disruption occurs at the following stage:

Explanation

A large number of components are required for translation (synthesis of a protein). These include all the amino acids that are found in the finished product, the mRNA to be translated, tRNA, functional ribosomes, energy sources and enzymes as well as protein factors needed for initiation, elongation and termination of the polypeptide chain. In eukaryotic cells, the ribosomes are either “free” in the cytosol or are in close association with the endoplasmic reticulum (which is then known as the “rough” endoplasmic reticulum or RER). The RER – associated ribosomes are responsible for synthesizing proteins that are to be exported from the cell as well as those that are destined to become integrated into plasma, endoplasmic reticulum or golgi membranes or incorporated into lysosomes.

188.

A 6-year-old child with suspected active tuberculous process has undergone diagnostic Mantoux test. What immunobiological preparation was injected?

Explanation

Tuberculin (Mantoux) skin test: this test is done by intradermal injection of tuberculoprotein (tuberculin), purified protein derivative (PPD). Type IV hypersensitivity reaction.

Immunization against tuberculosis is induced by injection of attenuated strains of bovine type of tubercle bacilli, Bacilli Calmette Guerin (BCG).

The Mantoux skin test should be read between 48 and 72hrs after administration. The basis of reading  is the presence or absence of induration, which may be determined by inspection and by palpation. A record should also be made of formation of vesicles, bullae, lymphangitis, ulceration and necrosis at the test site. The formation of vesicles, bullae or necrosis at the test site indicates positive result. A negative mantoux result usually signifies that the individual has never been exposed to Mycobacterium tuberculosis i.e. absence of cell mediated immunity to tuberculin.

189.

During postembryonal haemopoiesis in red bone marrow the cells of one of the cellular differons demonstrate gradual decrease in cytoplasmic basophilia as well as increase in oxyphilia, the nucleus is being forced out. Such morphological changes are typical for the following haemopoiesis type:

Explanation

    Proerythroblast → basophilic erythroblast → polychromatophilic erythroblast → orthochromatophilic erythroblast (normoblast) → polychromatophilic erythrocyte (reticulocyte) → Erythrocyte (red blood cell).

The orthochromatophilic erythroblast (normoblast) has a small, compact, densely stained nucleus. The cytoplasm is eosinophilic because of the large amount of hemoglobin. It is only slightly larger than a mature erythrocyte. It loses its nucleus  by extruding it from the cell, it is then ready to pass into a blood sinus of the red bone marrow.

IMG_9922

190.

When taking exams students often have dry mouth. The mechanism that causes this state results from the following reflexes:

Explanation

Reflex activity is the response to a peripheral nervous stimulation that occurs without our consciousness. It can be:

* Inborn or unconditioned: present from birth, does not require previous learning or training.

* Acquired or conditioned: developed after conditioning or training. Acquired after birth.

It is conditioned because the students have had a previous experience with examinations before. The manifestation of dry mouth is a sympathetic effect.

191.

A 67-year-old patient with clinical diagnosis of chronic bronchitis, pneumosclerosis, and cardiopulmonary decompensation has the biopsy material taken from the suspicious area in his right bronchus mucosa. Cellular and tissue atypism along with pearly bodies can be histologically detected. What pathologic process is characterized by the described histological changes?

Explanation

Squamous cell (epidermoid) carcinoma: these tumors usually arise in a large bronchus and are prone to massive necrosis and cavitation. The tumor is  diagnosed microscopically by identification of either intercellular bridges or keratinization. Usually the spread of squamous cell carcinoma is more rapid than the other histologic types. Frequently, the edge of the growth and the adjoining uninvolved bronchi show squamous metaplasia, epithelia dysplasia and carcinoma in situ.
192.

A microspecimen of heart shows rectangular cells from 50 to 120 micrometers large with central position of nucleus and developed myofibrils. The cells are connected by intercalated discs. These cells are responsible for the following function:

Explanation

     Cardiac muscle has the same types and arrangement of contractile filaments as skeletal muscle. Cardiac muscle fibers exhibit densely staining cross bands called intercalated disks, that cross the fibers in a linear fashion or frequently in a way that resembles the risers of a stairway. The intercalated disks represent highly specialized attachment sites between adjacent cells. The cardiac muscle nucleus lies in the center of the cell which is one feature that helps distinguish them from multinucleated skeletal muscle fibers, whose nuclei lie immediately under the plasma membrane. Cardiac muscle cells exhibit a spontaneous rhythmic contraction.

193. Untrained people often have muscle pain after sprints as a result of lactate accumulation. This can be caused by intensification of the following biochemical process:

Explanation

Lactate, formed by the action of lactate dehydrogenase (converting pyruvate to lactate) is the final product of anaerobic glycolysis in eukaryotic cells. In organs or cells that are poorly vascularized and/or lack mitochondria, formation of lactate is the major fate of pyruvate as seen in lens, cornea of the eye, kidney medulla, testes, leukocytes and red blood cells.

Aerobic glycolysis progresses to citric acid cycle from pyruvate. The cycle occurs totally in the mitochondria.

       Athletes that are exercising intensely for the short periods of time, such as in a sprint race, build up large amounts of lactate in their muscles as the result of anaerobic glycolysis. The “warming down” period of continual movement under aerobic conditions performed by athletes for approximately 15mins after a race increases circulation and removes lactate from the muscles.

194.

Poisoning caused by botulinum toxin that prevents calcium ions from entering axon nerve endings of motoneurons is life-threatening because it can lead to:

Explanation

Clostridium species are Gram-positive bacilli that grow under anaerobic conditions and produce spores that are present in the soil e.g. C. perfringens; C. tetani; C. botulinum; C. difficile.  Botulism is caused by exposure to a neurotoxin (popularly known as Botox) that is produced by the anaerobic Gram-positive organism Clostridium botulinum. Botox acts by blocking the release of acetylcholine from presynaptic neurons. C. botulinum grows in inadequately cooked foods and causes flaccid paralysis of respiratory and skeletal muscles. Resulting in respiratory arrest.
195.

Increased HDL levels decrease the risk of atherosclerosis. What is the mechanism of HDL anti-atherogenic action?

Explanation

krushkrok No13a (2014)

High density lipoprotein (HDL): is the antiatherogenic lipoprotein fraction (nicknamed the ‘good lipoprotein’).  High density lipoprotein (HDL) – transports cholesterol from periphery to liver.

Low density lipoprotein (LDL): is the proatherogenic lipoprotein fraction (nicknamed the ‘bad lipoprotein’). Low Density Lipoprotein (LDL) - transports cholesterol from liver to tissues.

FullSizeRender (9)

LCAT (lecithin-cholesterol acyltransferase) – catalyzes esterification of cholesterol

Cholesterol ester transfer protein (CETP) -  mediates transfer of cholesterol esters to other lipoprotein particles.

196. It has been found out that one of a pesticide components is sodium arsenate that blocks lipoic acid. Which enzyme activity is impaired by this pesticide?

Explanation

Pyruvate dehydrogenase complex: Mitochondrial enzyme complex linking glycolysis and citric acid cycle. The complex contains 3 enzymes that require 5 cofactors: Vit B1, B2, B3, B5, lipoic acid. Pyruvate → Acetyl-CoA.

The complex is similar to the α-ketoglutarate dehydrogenase complex (same cofactors, similar substrate and action), which converts α-ketoglutarate → Succinyl-CoA (CAC or TCA cycle). Arsenic inhibits lipoic acid. Findings: vomiting, rice-water stools, garlic breath.

197.

A drycleaner’s worker has been found to have hepatic steatosis. This pathology can be caused by the disruption of synthesis of the following substance:

Explanation

Hepatic steatosis can occur when humans are deprived of choline.

Choline + Phosphatidic acid → Phosphatidylcholine (lecithin, PC). In the liver PC can also be synthesized from phosphatidylserine (PS) and phosphatidylethanolamine (PE), when free choline levels are low, because it exports significant amounts of PC in bile and as a component of serum lipoproteins (needed for fat metabolism)

PS → PE →→→ PC. 3 methylation reactions between PE and PC. S-adenosylmethionine is the methyl group donor. If choline, phosphatidylcholine or methionine is deficient, there will be abnormal phospholipid synthesis, oxidative damage caused by mitochondrial dysfunction, lipoprotein secretion (remember, if VLDL cannot be secreted it will be accumulated & cause fatty liver degeneration as seen in hepatic steatosis). PC is also a major lipid component of lung surfactant.

198.

A 35-year-old man with peptic ulcer disease has undergone antrectomy. After the surgery secretion of the following gastro-intestinal hormone will be disrupted the most:

Explanation

image GRP - gastrin releasing peptide; GIP - gastric inhibitory peptide; ACh - acetylcholine The body of stomach ends in antrum. Junction between body and antrum is marked by an angular notch called Incisura angularis. Antrum is continued as the narrow canal, which is called pyloric canal or pyloric end. Pyloric canal opens into first part of small intestine called duodenum. Gastrin: source - G cells (antrum of stomach, duodenum) Cholecystokinin (CCK): source - I cells (duodenum, jejunum) Secretin: source - S cells (duodenum) Histamine: source - Enterochromaffin-like (ECL) cells [NB: Antrectomy is the resection of the antrum of the stomach.]
199.

Characteristic sign of glycogenosis is muscle pain during physical work. Blood examination usually reveals hypoglycemia. This pathology is caused by congenital deficiency of the following enzyme:

Explanation

    Glycogen phosphorylase is the rate-determining enzyme in Glycogenolysis (break down of glycogen). Glycogen phosphorylase cleaves glycogen to glucose 1-phosphate (first step in glycogenolysis). This pathway mobilizes stored glycogen in liver to replenish used glucose. It also breaks down glycogen in muscle to glucose, to produce energy during physical work. If this enzyme is deficient, then used glucose cannot be replaced leading to hypoglycemia.

In lysosomal glycosidase, there is still normal blood sugar levels (no hypoglycemia). α-amylase and ɣ-amylase is involved in digestion of polysaccharides. Glucose 6-phosphate dehydrogenase (G6PD) is the rate-limiting enzyme in pentose phosphate pathway which catalyzes an irreversible oxidation of glucose 6-phosphate to 6-phosphogluconolactone.

200.

Histologic specimen of a kidney demonstrates cells closely adjoined to the renal corpuscle in the distal convoluted tubule. Their basement membrane is extremely thin and has no folds. These cells sense the changes in sodium content of urine and influence renin secretion occurring in juxtaglomerular cells. Name these cells:

Explanation

     image

Juxtaglomerular apparatus is formed by 3 different structures:

·        Macula densa: is the end portion of thick ascending segment as it opens into the distal convoluted tubule. It is situated between the afferent and efferent arterioles of the same nephron. It is very close to afferent arteriole. Macula densa is formed by tightly packed cuboidal epithelial cells.

·        Mesangial cells: are situated in the triangular region bound by afferent arteriole, efferent arteriole and macula densa.

·        Juxtaglomerular cells: are specialized smooth muscle cells situated in the wall of afferent arteriole just before it enters the Bowman’s capsule.